! Крок 1 Фарм 2007-2018 Іноз Англ
! Крок 1 Фарм 2007-2018 Іноз Англ
1
krok123.in.ua
Krok 1 Pharmacy 2007 2
14. According to Hardy-Weinberg law 20. In order to keep vitality and stabili-
coagulating effect of coagulant ion is ty of eubiotics microorganisms in frozen
influenced by: state are dried under conditions of high
vacuum. What method is it?
A. Ion charge
B. Ion size A. Lyophilization
C. Adsorbability B. Pasteurization
D. Hydratability C. Tyndallization
E. Polarizability D. Inactivation
E. Hybridization
15. Name the type of reaction applied for
detection of F e3+ cation: 21. Hydrogen is characterized by the
following oxidation rates: −1; 0; +1. The
A. Complexing −1 oxidation rate hydrogen has in:
B. Precipitation
C. Hydrolysis A. Hydrides
D. Neutralization B. Acids
E. Renewing C. Hydroxides
D. Water
16. Pharmaceutists widely apply drugs in E. Acid salts
form of colloid-disperse systems. What
method of sol production is rated as physi- 22. What compounds entered into a reacti-
cal condensation? on if its products were nitrobenzene and
water:
A. Solvent substitution
B. Reduction
C. Oxidation
D. Hydrolysis
E. Double exchange
17. A patient was diagnosed with anacydic
gastritis. What enzyme activity will be
reduced?
2
krok123.in.ua
Krok 1 Pharmacy 2007 3
A.
A. Index of solution refraction
B. Angle of rotation of plane of polarized
light
C. Angle of total internal reflection of light
B. ray
D. Light ray angle
E. Angle of light refraction
27. In order to estimate antibiotic suscepti-
C. bility of a patient doctors introduced
him intracutaneously 0,2 ml of penicilli-
ne solution. Ten minutes after introducti-
on there appeared hyperemy and edema.
D. What type does this reaction relate to
(according to Coomb’s and Gell’s classi-
fication)?
A. Anaphylactic reaction
E. B. Cytotoxic reaction
C. Reaction of Arthus phenomenon type
D. Delayed-type hypersensitivity
E. Tuberculine reaction
28. Name inhibitory transmitters:
23. What disease of blood coagulation
system is based upon abrupt decelerati- A. GABA and glycin
on of blood coagulation due to disturbed B. Adrenaline and noradrenaline
formation of plasma thromboplastin (VIII C. Noradrenaline and dopamine
factor deficit)? D. Serotonin and glycin
E. Acetylcholine and GABA
A. Hemophilia
B. Thrombocytopenic purpura 29. Quantitative determination of
C. Hemorrhagic vasculitis pharmaceutical substances can be done
D. Symptomatic thrombocytopenia by method of acidimetry. Its titrant is the
E. Hemorrhagic purpura secondary standard solution of chloride
acid. What compound helps to determi-
24. To the membrane proteins that contact ne the precise concentration of chloride
with this or that biologically active acid?
substance transmitting information into
the cell belong: A. Sodium tetraborate
B. Oxalate acid
A. Receptor proteins C. Potassium dichromate
B. Pump proteins D. Sodium thiosulfate
C. Enzyme proteins E. Magnesium sulfate
D. Channel proteins
E. Glycocalix 30. Hop sprouts wind around a support
and climb upwards. That means that they
25. What segment of digestive tract are:
secretes digestive juice that has acid
reaction? A. Creeping
B. Recumbent
A. Stomach C. Arrect
B. Oral cavity D. Tenent
C. Small intestine E. Trailing
D. Large intestine
E. Esophagus 31. A patient was prescribed with
bile preparation for the purpose of
26. Concentration of ethyl alcohol in some improvement of rich food digestion. What
drug formulations and tinctures can be components of this preparation take part
determined by means of refractometry. in fat emulsification?
For this purpose the following characteri-
stic is measured:
3
krok123.in.ua
Krok 1 Pharmacy 2007 4
A. Bile acids
B. Cholesterol and its ethers A. Convection
C. Diglyceride B. Evaporation
D. Bilirubin-glucuronids C. Heat radiation
E. Higher fatty acids D. Heat conduction
E. Heat radiation and heat conduction
32. What carbon atoms in the given
compound are in the second valence state 37. What is molecular weight of a gas if its
of (sp2 -hybridization)? hydrogen density is 15?
A. 30 g/mole
B. 7,5 g/mole
C. 15 g/mole
D. 45 g/mole
E. 60 g/mole
A. 1 and 2 38. During an exam arterial pressure
B. 1 and 3 of a student rose and his heartbeat
C. 2 and 3 accelerated. Name the probable cause of
D. 3 and 4 this phenomenon:
E. 5 and 6
A. Increase of tonus of sympathetic
33. According to the requirements of nervous system
WHO and Pharmacopoeia different drug B. Extension of excitability threshold α
dosage forms of unsterile preparations and β adrenoreceptors
are allowed to have a certain quantity C. Increase of circulating blood volume
of bacteria and fungi. What quantity of D. Decrease of tonus of parasympathetic
saprophytic bacteria and fungi in 1 g (ml) nervous system
of a peroral preparation will ensure its E. Excretion of glucocorticoids
safety?
39. Titrant of chelatometry method is
A. 1000 bacteria and 100 mold fungi trilon B solution that forms complex
B. 500 bacteria and 50 mold fungi compounds with metal cations irrespecti-
C. 250 bacteria and 25 mold fungi ve of their valence at a ratio of:
D. 500 bacteria and 200 mold fungi
E. 1500 bacteria and 150 mold fungi A. 1 : 1
B. 1 : 3
34. It is known that digestion of proteins, C. 1 : 2
fats and carbohydrates happens due to D. 2 : 1
protease, lipase and amylase respectively. E. 3 : 1
What digestive juice contains all three
enzyme groups enough for digestion? 40. Name the type of reaction that takes
place during detection of ascorbic acid in
A. Juice of pancreas a preparation by iodometric method:
B. Saliva
C. Gastric juice A. Oxidation-reduction
D. Bile B. Acylation
E. Juice of large intestine C. Neutralization
D. Precipitation
35. Presence of chlorides in drinking water E. Complexing
can be detected by method of mercuri-
metry. One of the following solutions is 41. During quantitative estimation of
used as titrant: glucose by polarimetric method the
following factor is measured:
A. Hg(NO3)2
B. Hg2 (NO3 )2 A. Angle of rotation of polarized beam
C. HgCl2 plane
D. HgSO4 B. Coefficient of light refraction
E. Hg2 Cl2 C. Rate of polarized beam absorption by a
solution
36. Under conditions of high temperature D. Beam dispersion by a solution
of the environment a fan eases being E. Optical density of a solution
in premises because it intensifies heat
transfer by means of: 42. An important role during starvati-
4
krok123.in.ua
Krok 1 Pharmacy 2007 5
5
krok123.in.ua
Krok 1 Pharmacy 2007 6
A. Adiabatic A. ∆G=0
B. Isothermal B. ∆F=0
C. Isochoric C. ∆H=0
D. Isobaric D. ∆U=0
E. Polytropic E. ∆S=0
52. Electrolyte solutions are medicinal 57. A patient had an attack of calculous
preparations. What is the maximum value cholecystitis that was accompanied by
of isotonic coefficient for MgSO4 soluti- saponated feces, steatorrhea. These
on? changes are the evidence of disturbance
of the following stage of lipometabolism:
A. 2
B. 4 A. Digestion and absorption
C. 3 B. Transport
D. 5 C. Intermediary metabolism
E. 7 D. Adipose tissue exchange
E. Depositing
53. Pulp of a needle leaf consists of living
tissue with internal ansiform outgrowths 58. Labels of some drugs have the followi-
of membrane. Along these outgrowths the ng inscription: "shake before use!". This
chloroplasts are placed. Name the type of warning is induced by:
this leaf’s parenchyma:
A. Sedimentation
A. Folded B. Coagulation
B. Spongy C. Solubility of disperse systems
C. Palisade D. Insolubility of disperse systems
D. Storage E. -
E. Aeriferous
59. Quantitative content of calcium chlori-
54. Bacteriological control of unsterile de can by measured by method of direct
drugs assumes the possibility of presence chelatometric titration. Choose an indi-
of some microorganism groups. What cator for registering the end point of ti-
groups are these? tration:
A. Sarcina A. Eriochrome black T
B. Colon bacillus B. Phenolphthalein
C. Blue pus bacillus C. Methyl red
D. Aurococcus D. Eosin
E. Hemolytic streptococcus E. Starch
55. Choose a reagent for production of an 60. A plant under examination has a
acetic acid hydrazide from ethyl acetate: storage root; its stems are ribbed and
channelled, hollow; leaves are many ti-
mes pinnatisected, leafstalk has a boot;
inflorescence is the compound umbel;
fruit is the cremocarp with essential oil
canaliculi in the pericarp. Such characteri-
stics are typical for the plants of the
following family:
A. H2 N NH2
B. NH3 A. Apiaceae
C. H2 N CH3 B. Solanaceae
D. C6 H5 NH2 C. F abaceae
E. C6 H5 NHNH2 D. Brassicaceae
E. Scrophulariaceae
56. What expression corresponds with
the state of chemical equilibrium under 61. One of fleshy fruits under exami-
constant pressure and temperature? nation is characterized by essential oil
exocarp, spongy mesocarp and overgrown
endocarp consisting of juice sacs. What
fruit was examined?
6
krok123.in.ua
Krok 1 Pharmacy 2007 7
A. Selivanov’s
B. Lugol’s
A. Electron-acceptor properties of ni- C. Fehling’s
trogen atom D. Chempure
B. Aromatic nature of pyridine nucleus E. Tollens’
C. Alkaline properties
D. Hybridization of carbon atoms 78. A smear from frothy and purulent
E. Cycle size vaginal discharges of a 42 y.o. woman was
stained by Romanovsky-Giemsa method.
73. Argentum nitrate solution was added Its analysis revealed some microorgani-
to a solution containing anions of the sms of flagellates class. What microorgani-
second analytical group. It resulted in ms were the most probably revealed?
generation of light yellow precipitate that
was unsoluble in nitric acid and partly A. T rihomonas vaginaslis
soluble in ammonium solution. What ani- B. Leishmania donovani
ons are contained in the solution? C. T rypanosoma gambiense
D. T rihomonas hominis
A. Bromide-ions E. Lamblia intestinalis
B. Iodide-ions
C. Chloride ions 79. During bacteriological examinati-
D. Sulfide-ions on of sputum of a child with choking
E. Arsenite-ions cough and fever there were revealed
glossy smooth colonies growing on casein-
74. It is known that bluish purple petal charcoal agar and reminding of mercury
coloration of a plant under examination drops. Microscopic examination revealed
varies up to pink or light pink accordi- short Gram-negative bacteria. What
ng to pH of cellular fluid of vacuole. It is microorganism was secured from the
caused by presence of: sputum?
A. Anthocyanins A. Bordetella pertussis
B. Carotins B. Haemophylus inf luenzae
C. Xanthophylls C. Corynebacterium dyphtheriae
D. Phycobilins D. Klebsiella pneumoniae
E. Chlorophylls E. Streptococcus pyogenes
75. Microscopic examination of a stem of 80. It was suspected that among workers
a perennial plant revealed integumentary of serum medications factory of regional
tissue of secondary origin that was formed hemotransfusion station there are carriers
as a result of activity of: of pathogenic aurococcus. What medium
should the material from nasopharynx of
A. Phellogen workers be inoculated of in order to reveal
B. Procambium aurococcous carriage?
C. Cambium
D. Pericycle A. Yolk-salt medium
E. Protoderm B. Endo agar
C. Beef-extract broth
76. Choose a reagent that can be used for D. Ressler’s medium
generation of propanol-2 from acetone: E. Blood agar
A. H2 81. According to Pharmacopoeia requi-
B. CH3 OH rements all drugs for topical administrati-
C. CH3 I on should be monitored for "microbi-
D. HCN ological purity". Detection of the followi-
E. HCOH ng microorganisms is the evidence of
77. Three enumerated test tubes contain uselessness of this drug group in medi-
solutions of glucose, fructose and starch. cal practice:
What reagent can help to detect fructose? A. Aurococci
B. Yeast fungi
C. Humicular staphylococci
D. Mold fungi
E. -
8
krok123.in.ua
Krok 1 Pharmacy 2007 9
A. Increased respiration rate and depth 88. Under certain conditions of qualitative
B. Decreased respiration rate and depth analysis K4 [F e(CN)6 ] is a specific reagent
C. Decreased respiration rate and to F e3+ cations. What colour is the preci-
increased respiration depth pitate?
D. Increased respiration rate and
decreased respiration depth A. Blue
E. Respiration would stay unchanged B. White
C. Brown
83. What class is represented by D. Red
nitroglycerine medication used for E. Black
stenocardia treatment?
89. During bacteriological examination of
A. Ester solutions prepared in a drug-store there
B. Nitrogen-containing alcohols appeared red colonies with metallic lustre
C. Ethers that grew on Endo agar. What bacteria
D. Nitroalkanes can they be?
E. Polyatomic alcohols
A. Escherichiae
84. What of the given compounds makes B. Shigellae
the reactions of electrophylic substitution C. Staphilococci
(SE ) the easiest? D. Streptococci
E. Salmonellae
A. Phenol
B. Toluol 90. In the technology of medicinal
C. Chlorobenzene preparation temperature and pressure are
D. Benzaldehyde sustained constant very often. What is this
E. Benzene sulfacid process called?
10
krok123.in.ua
Krok 1 Pharmacy 2007 11
12
krok123.in.ua
Krok 1 Pharmacy 2007 13
15
krok123.in.ua
Krok 1 Pharmacy 2007 16
A. Vasopressin A. Ozone
B. Adrenaline B. Phosgene
C. Thyroxine C. Quartz
D. Aldosterone D. Corundum
E. Noradrenaline E. Diamond
18
krok123.in.ua
Krok 1 Pharmacy 2007 19
A. F A. α-amylase
B. C B. Phosphatase
C. Si C. Enterokinase
D. P D. Chemotrypsin
E. I E. Lysozyme
196. Optic isomerism is possible for one of 199. Solution of a medicinal preparati-
the following compounds: on under examination contains cations
of magnesium (II) and aluminium (III).
A. Iodofluorochloromethane (CHJF Cl) What reagent will help to separate these
B. Methane (CH4 ) cations during analysis of this preparati-
C. Chloroform (CHCl3 ) on?
D. Dichloromethane (CH2 Cl2 )
E. Tetrachloromethane (CCl4 ) A. Alkali solution
B. Hydrogen peroxide in acidic medium
197. Stability of concentrated emulsi- C. Argentum nitrate solution
ons can be increased by adding surface- D. Hydrogen peroxide in ammoniac medi-
action substances and high-molecular um
compounds that are: E. Chloride acid solution
A. Emulsifiers 200. What method ensures reliable sterili-
B. Activators zation of biological liquids (serums, soluti-
C. Catalysts ons, enzymes, vitamines etc.) that cannot
D. Solvents be exposed to high temperatures?
E. Absorbents
A. Tyndallization
198. Pancreas secretes an enzyme that B. Dry heat
is aible to break up α − 1,4-glycosidic C. Flowing steam
linkages in a glycogen molecule. What D. Moist steam under pressure
enzyme is it? E. Flaming
19
krok123.in.ua
Krok 1 Pharmacy 2008 1
on has tetraquetrous stalk, flowers with disperse systems are widely applied in
bilabiate corolla, coenobium fruit. These pharmaceutical practice. What method
characteristics are typical for the followi- of sol production relates to physical
ng family: condensation?
A. Lamiaceae A. Solvent substitution
B. Papaveraceae B. Reduction
C. Polygonaceae C. Oxidation
D. Solanaceae D. Hydrolysis
E. Scrophulariaceae E. Double exchange
13. Name a complex compound that has 19. A child with evident hypotrophy got
antitumoral activity: edemata on his lower extremities, asci-
tes. What is the main mechanism of
A. [P t(NH3 )2 Cl2 ] pathogenesis of cachectic edema?
B. [Co(NH3 )5 NO3 ]Cl2
C. Na4 [Sn(OH)3Cl3 ] A. Drop of oncotic pressure of blood
D. [Cu(NH3 )4 (SCN)2 ] plasma
E. K2 Na[Co(NO2 )6 ] B. Rise of hydrostatic blood pressure
C. Rise of oncotic pressure of intercellular
14. Solution applied as isotonic soluti- fluid
on should have the following osmotic D. Increased permeability of vascular wall
pressure: E. Disturbance of lymph outflow
A. 700 - 800 kPa 20. A patient has impaired mesopic vision,
B. 200 - 300 kPa his photopic vision is normal. What is the
C. 300 - 400 kPa probable cause of such vision anomaly?
D. 500 - 600 kPa
E. 900 - 1000 kPa A. Vitamin A deficiency
B. Hyperopia
15. According to Schultze-Hardy rule C. Cones disfunction
coagulating action of coagulant ion is D. Myopia
affected by: E. Vitamin D deficiency
A. Ion charge 21. Thiocyanatometry is based upon using
B. Ion size of secondary standard solution of potassi-
C. Adsorbability um thiocyanate that should be standardi-
D. Hydratability zed according to the following standard
E. Polarizability solution of:
16. What reaction is applied for detection A. Silver nitrate
of F e3+ cation? B. Hydrochloric acid
C. Sulfuric acid
A. Complexing D. Iron (II) sulfate
B. Precipitation E. Copper (II) nitrate
C. Hydrolysis
D. Neutralization 22. Concentration of potassium di-
E. Reduction chromate in a solution was determined
by means of iodometry. Name a titrant of
17. Dimethyl glyoxime entered into reacti- iodometric method for determination of
on with a solution that contained cati- strong oxidizer:
ons of the IV analytical group (acid-base
classification). The deposition turned cri- A. Sodium thiosulfate
mson. What cation caused this analytical B. Sodium hydroxide
effect? C. Potassium iodide
D. Potassium permanganate
A. Nickel cation (II) E. Potassium bromate
B. Mercury cation (II)
C. Copper cation (II) 23. Solution of potassium chromate was
D. Cadmium cation (II) added to a solution under examination.
E. Cobalt cation (II) As a result of it some yellow deposition
settled down. This deposition cannot be
18. Drugs in form of colloid- dissolved in acetic acid. This means that
21
krok123.in.ua
Krok 1 Pharmacy 2008 3
45. During sanitary and bacteriological limbs is increased, their spinal reflexes
examination of air in a drugstore it was are intensified, reflex zones are increased.
revealed that the air had high concentrati- What type of CNS disorder is it?
on of sanitary meaningful microorgani-
sms. What microorganisms are these? A. Central paralysis
B. Peripheral paralysis
A. Staphylococcus aureus and hemolytic C. Spinal shock
streptococcus D. Atonic paralysis
B. Diphtheritic and tuberculous bacilli E. Reflex paralysis
C. Colibacilli and blue pus bacilli
D. Epidermal staphylococcus and Sarcina 51. Bacteriological control of unsterile
E. Enterococci and Citrobacter drugs allows presence of small quantity of
some microorganism groups. What group
46. Steroid hormones are synthesi- is meant?
zed out of a precursor that contai-
ns cyclopentanoperhydrophenanthrene. A. Sarcina
Name this precursor: B. Colon bacillus
C. Bacillus pynocyaneus
A. Cholesterol D. Staphylococcus aureus
B. Acetyl-CoA E. Hemolytic streptococcus
C. Malonyl-CoA
D. Levulinic acid 52. As a result of reduced water
E. Tyrosine reabsorption in nephron tubules daily di-
uresis of a patient has increased up to 10
47. In the pharmaceutical production litres. This might be caused by reduced
processes of drug synthesis take place secretion of the following hormone:
under different conditions. Entropy stays
unchanged in the following process: A. Vasopressin
B. Aldosterone
A. Adiabatic C. Parathormone
B. Isothermal D. Thyrocalcitonin
C. Isochoric E. Insulin
D. Isobaric
E. Polytropic 53. A product of complete acetylation of
glycerine relates to the following class of
48. Pulp of a needle leaf consists of living organic compounds:
tissue with inner ansiform protuberances
of membrane and chloroplasts along A. Ester
them. What is type of this leaf’s B. Ether
parenchyma? C. Ketone
D. Acetal
A. Plicate E. Phenol
B. Spongioid
C. Palisade 54. Name a product of ester condensation
D. Storage of acetaldehyde (Tishchenko reaction):
E. Aeriferous
49. A patient suffering from gastric ulcer
for a long time has dramatic emaciation,
skin pallor, appetite loss, aversion to meat
products. Biopsy of mucous membrane
of stomach revealed cellular atypia. What A. Ethyl acetate
pathology are these symptoms typical for? B. Acetone
C. Crotonic aldehyde
A. Malignant tumour of stomach D. Malonic acid
B. Benign tumour of stomach E. Acetoacetic aldehyde
C. Polyposis
D. Hypertrophic gastritis 55. Water-soluble vitamins take coenzyme
E. Helminthic invasion form in an organism. Thiamine di-
phosphate is coenzyme of the following
50. A patient had cerebral haemorrhage vitamin:
that made impossible active motions of
left arm and leg. Muscle tone of these
24
krok123.in.ua
Krok 1 Pharmacy 2008 6
75. A few minutes afer repeated cardiac insufficiency has got soft ti-
introduction of penicillin a patient got ssue edemata on his shins. What is the
dyspnea, tongue numbness, hyperemia leading pathogenetic factor of edema
and then skin pallor. The patient also lost development?
consciousness. What is the cause of such a
grave condition? A. Rise of hydrostatic pressure in capillari-
es
A. Anaphylactic shock B. Drop of osmotic pressure in blood
B. Serum sickness plasma
C. Hemolytic anemia C. Rise of oncotic pressure in tissues
D. Acute glomerulonephritis D. Drop of hydrostatic pressure in capi-
E. Bronchial asthma llaries
E. Rise of osmotic pressure in tissues
76. Cations of the third analytical group
(acid-base classification) can be isolated 81. It is known that depending on pH
in course of systematic analysis by means of cellular fluid petal coloration can vary
of the following group reagent: from blue-and-violet to pink and light pi-
nk. This is caused by presence of:
A. 1 M solution of sulfate acid in presence
of ethanol A. Anthocyanins
B. 1 M solution of potassium chromate B. Carotins
C. 0,1 M solution of sodium carbonate C. Xanthophylls
D. 0,1 M solution of ammonium oxalate D. Phycobilins
E. 1 M solution of ammonium carbonate E. Chlorophylls
77. Solution of potassium iodide was 82. Which of the following plants has
added to the solution acidated with sulfate pome fruit?
acid that contained anions of the third
analytical group. Release of free iodine A. Sorbus aucuparia
is observed. What anion are present in the B. Prunus domestica L.
solution? C. Amygdalus communis
D. Rosa majalis
A. Nitrite ion E. Prunus padus
B. Carbonate ion
C. Sulfate ion 83. Bacteriological examination for
D. Bromide ions bacteria carrying of drugstore workers
E. Acetate ions revealed that one of the pharmacists
had bacteria of genus Staphylococcus.
78. During gravimetric determination What morphological pecularities of mi-
of mass fraction of sulfate ions in the crobal cell arrangement are typical for this
magnesium sulfate preparation precipi- genus?
tation is performed by means of bari-
um chloride solution. Precipitated barium A. They are arranged in form of bunch of
sulfate should be rinsed with: grapes
B. They are arranged in form of a chain
A. Diluted solution of sulfate acid C. They are arranged isolatedly
B. Distilled water D. They are arranged in pairs
C. Solution of barium chloride E. They are arranged in tetrads
D. Solution of sodium sulfate
E. Solution of hydrochloride acid 84. Crude herbal drugs must be examined
for yeastlike fungi. What agar can ensure
79. A patient has bradycardia, moderate development of these microorganisms so
hypotension, decrease of basal metaboli- that associating microflora will grow very
sm, edemata. What abnormality can slowly or won’t grow at all?
induce such syndrome?
A. Sabouraud’s peptone agar
A. Thyroid hypofunction B. Endo agar
B. Parathyroid hypofunction C. Meat infusion agar
C. Thyroid hyperfunction D. Milk-salt agar
D. Parathyroid hyperfunction E. Blood agar
E. Adrenal hypofunction
85. A patient took maximally deep breath.
80. A patient suffering from chronic Air volume being in lungs under these
27
krok123.in.ua
Krok 1 Pharmacy 2008 9
D. A.
B.
E.
C.
A. Gluconeogenesis A. Ascorutinum
B. Glycogenolysis B. Thiamine hydrochloride
C. Oxidation of fatty acids C. Cyanocobalamin
D. Ketogenesis D. Nicotinic acid
E. Glycolysis E. Pyridoxine hydrochloride
103. An electrode composed by scheme 109. What form of hypoxia develops duri-
Au3+ | Au relates to the following type: ng shock and collapse?
A. I type electrodes A. Circulatory
B. II type electrodes B. Respiratory
C. III type electrodes C. Hypoxic
D. Oxidation-reduction electrodes D. Hemic
E. Ion-selective electrodes E. Tissue
104. You need to prepare ammoniac 110. Employees of a physicochemical
buffer solution. For this purpose you laboratory prepared water solutions of
should add the following solution to the urea, glucose, sodium sulfate, aluminium
water solution of ammonia: sulfate and sodium benzoate all of which
had the same molar concentration. What
A. Solution of ammonium chloride solution has the highest osmotic pressure
B. Solution of chloride acid under 298o K?
C. Solution of sulfate acid
D. Solution of potassium chloride A. Aluminium sulfate
E. Solution of sodium sulfate B. Urea
C. Glucose
105. Alkadiene is a name for aliphatic D. Sodium benzoate
carbohydrates with double bonds. Choose E. Sodium sulfate
a general formula for homologous series
of alkadienes: 111. Potassium dichromate K2 Cr2 O7 is
applied as oxidant in acidic medium. What
A. Cn H2n−2 is the product of reduction of dichromate-
B. Cn H2n+2 ion Cr2 O72− under these conditions?
C. Cn H2n
D. Cn H2n+ 1 A. Cr3+
E. Cn H2n−1 B. Cr(OH)3
C. Cr(OH)2
106. Urea is a derivative of carbonic acid. D. [Cr(OH)6]3−
Choose a denomination of urea: E. Cr2 O3
A. Diamide of carbonic acid 112. Blood analysis revealed rise of acti-
B. Monoamide of carbonic acid vity of LDH1 , LDH2 , aspartate ami-
C. Ethylic ether of carbamic acid notransferase, kreatine phosphokinase-
D. Diethylic ether of carbonic acid MB. Biochemical disorder is observed in
E. Dimethylic ether of carbonic acid the following organ:
107. Choose a pair of substances that A. Heart
can be used for standardization of 0,1 M B. Skeletal muscles
solution of KMnO4 : C. Kidneys
A. Na2 C2 O4 , H2 C2 O4 D. Liver
B. K2 CO3 , CH3 COOH E. Pancreas
C. CH3 COOK, H2 C2 O4 113. Limited swelling of gelatine comes to
D. KHC2O4 , HCOOH
E. Na2 C2 O4 , CH3 COOH unlimited one (solution formation) under
the following conditions:
108. A patient complains about gingival
haemorrhage, petechial haemorrhages. A. Heating
What vitamin preparation should be B. Cooling
recommended? C. In presence of SO42 ions
D. In presence of Cl ions
E. If pH medium matches with isoelectric
point
30
krok123.in.ua
Krok 1 Pharmacy 2008 12
114. What compound will be produced 119. You are given 0,05 M solution of
during reduction of methyl ethyl ketone? versene. What is standard substance for
standardization of this solution?
A. Metallic zinc
B. Sodium tetraborate
C. Sodium hydroxide
D. Oxalic acid
E. Potassium dichromate
A. secondary-butyl alcohol
B. Butanol-1 120. A 56 year old patient complains
C. Isobutyl alcohol about limitation of movements and pain
D. tertiary-butyl alcohol in hand joints, mainly at night. Objecti-
E. Propanol-2 vely: there is a disfiguring painful swelling
of affected joints. Blood and urine have
115. A female patient has been treated wi- high concentration of uric acid. What di-
th antibiotics for a long time. Thereafter sease has developed?
examination of smears form vaginal
secretion revealed oval cells with well- A. Gout
defined nucleus, some cells gemmate. B. Pellagra
What preparations can help to confirm C. Phenylketonuria
the diagnosis "candidosis"? D. Alkaptonuria
E. Tyrosinosis
A. Antifungal
B. Antibacterial 121. Electronic microscopy of a cell
C. Antichlamydial revealed mitochondrial destruction. What
D. Antiviral processes are disturbed?
E. Antiprotozoal A. ATP synthesis
116. One of mass production drugs is B. Protein biosynthesis
produced by inactivation of bacterial C. Glycolysis
exotoxin by formalin. What is this drug D. Synthesis of nucleic acids
for? E. Fat synthesis
31
krok123.in.ua
Krok 1 Pharmacy 2008 13
32
krok123.in.ua
Krok 1 Pharmacy 2008 14
A. Gastrin A. HCl
B. Vasointestinal polypeptide B. HCN
C. Neurotensin C. H2 CO3
D. Somatostatin D. CH3 COOH
E. Insulin E. H2 SO3
152. What denomination corresponds wi- 157. What is oxidation number of the
th the given formula? central atom in the compound H[AuCl4 ]?
A. +3
B. 0
C. +1
D. +2
E. +4
A. Benzo [b] pyridine 158. A child got burn on his hand caused
B. Benzo [b] pyrone-4 by hot water. Burn skin is bright red. What
C. Imidoazopyrimidine disturbance of local blood circulation is it?
D. Pyrazinopyrimidine
E. Benzothiazole A. Arterial hyperemia
B. Venous hyperemia
153. According to the IUPAC C. Stasis
nomenclature the given compound has D. Thrombosis
the following denomination: E. Embolism
159. A plantation of medicinal plants was
affected by a disease that caused yellow
spots and necrotic areas on leaves. Jui-
ce of affected plants remains infectious
even after bacterial filtration but after its
A. 1, 2, 3 − propanetriol inoculation on cultural medium growth of
B. 1 − propanol causative agent wasn’t observed. Causati-
C. 2 − propanol ve agent of this disease relates most
D. 1 − propanethiol probably to the following group of plant
E. 1, 2 − propanediol pathogenic microorganisms:
154. A 13 year old child complains A. Viruses
about poor appetite, pain in the right B. Fungi
subcostal area. Microscopical examinati- C. Actinomycetes
on of duodenal contents revealed big D. Bacteria
pyriform cells with two nuclei. What mi- E. Mycoplasma
croorganism was revealed?
160. It is known that a peroral drug contai-
A. Lamblia ns over 1 billion of living microbal cells
B. Trichomonad per 1 millilitre. Nonetheless the drug was
C. Amoeba accepted as applicable. What drug group
D. Trypanosoma does it relate to?
E. Toxoplasma
A. Eubiotics
155. Systems relate to colloid-disperse B. Antibiotics
ones if size of their particles is within the C. Vitamins
following range: D. Sulfanilamides
E. Immunostimulants
A. 10−9 – 10−7 m
B. 10−7 – 10−4 m 161. Microscopical examination of a leaf
C. > 10−4 m revealed water stomata on its serration.
D. ≶ 10−9 m These stomata are for exudation of liquid-
E. 10−9 – 10−4 m drop moisture. This process is called:
156. 0,1 M solution of the following acid
has the highest concentration of hydrogen
ions:
35
krok123.in.ua
Krok 1 Pharmacy 2008 17
A. Guttation A. Ischemia
B. Gas exchange B. Thrombosis
C. Internal secretion C. Embolism
D. Transpiration D. Arterial hyperemia
E. Photosynthesis E. Venous hyperemia
162. A man has symptoms of cardi- 167. Choose an appropriate indicator for
ovascular atherosclerosis. The most fixation of titration end point in method
probable characteristic of this state will of bromatometry:
be growth of the following biochemical
value: A. Methyl red
B. Phenolphthalein
A. Concentration of low-density li- C. Starch
poproteins D. Methyl blue
B. Concentration of high-density li- E. Tropeolin 00
poproteins
C. Concentration of chylomicrons 168. Decarboxylation of 5-hydroxytryptophane
D. LDH5 activity gives origin to a certain biogenic ami-
E. Activity of pancreatic lipase ne with vasoconstrictive action. What bi-
ogenic amine is it?
163. Presence of the following ion of d-
elements in solutions can be exploited by A. Serotonin
means of K4 [F e(CN)6 ]: B. Histamine
C. Gamma-aminobutyric acidc
A. F e3+ D. Putrescine
B. Zn2+ E. Cadaverine
C. Cr 3+
D. Ni2+ 169. An annual plant of the Asteraceae
E. Cu2+ family has tripartite leaves, apical
anthodia with tubular flowers, flat
164. Sol is one of drug forms. What achenocarps that are tenent due to 2-3
happens if sols are fused with oppositely bristly serratures. This plant is:
charged granules?
A. Bidens tripartita
A. Mutual coagulation B. Chamomilia recutita
B. Thixotropy C. Centaurea cyanus
C. Sedimentation D. Echinacea purpurea
D. Contraction E. Artemisia vulgaris
E. Lyophilization
170. Crop production includes cultivation
165. A patient has obstructive respiratory of medicinal essential oil plants that don’t
failure. Name a disease that is usually grow in Ukraine wildely, namely Mentha
accompanied by such type of respiratory piperita, Ortosiphon stamineus, and also:
failure:
A. Salvia officinalis
A. Bronchial asthma B. Origanum vulgare
B. Pneumonia C. Leonurus cardiaca
C. Exudative pleuritis D. Thymus serpyllum
D. Pneumoconiosis E. Leonurus quinquelobatus
E. Pneumothorax
171. A patient was admitted to the infecti-
166. A patient is 54 year old. After intense ous department of a hospital. His provisi-
emotional stress he felt strong pain behi- onal diagnosis was "acute gastroenteritis".
nd his breastbone irradiating to his left Inoculation of feces on bismuth-sulfite
arm and left part of his neck. He felt also agar induced growth of black colonies wi-
death anxiety and broke into a cold sweat. th metallic glitter. What microorganisms
Nitroglycerine relieved pain. Name a di- should you think of?
sturbamce of local blood circulation in
heart that has developed in this case: A. Salmonellae
B. Escherichia
C. Shigella
D. Yersinia
E. Brucella
36
krok123.in.ua
Krok 1 Pharmacy 2008 18
D.
E.
37
krok123.in.ua
Krok 1 Pharmacy 2008 19
A. Indole
B. Bilirubin A. Proline hydroxylase
C. Lactate B. Alanine aminotransferase
D. Kreatine C. Pyruvate carboxylase
E. Glucose D. Tryptophane hydroxylase
E. Glutaminase
177. A patient takes blocker of muscari-
nic cholinoreceptors of parasympathetic 183. Low rate of vitamin B6 in the dietary
nerve organ synapses. What changes of intake leads to disturbance of protein
heart activity will be observed? metabolism. What biochemical processes
in the patient’s organism will become less
A. Heart rate rise active?
B. Heart rate and heart force fall
C. Heart rate fall A. Transamination
D. Heart force fall B. Reduction-oxidation
E. Prolongation of atrioventricular delay C. Phosphorilation
D. Methylation
178. Anxious condition can be characteri- E. Hydrolysis
zed by reduced salivation and sense of
dry mouth. What mediator is exuded out 184. During blowing up a baloon a boy
of nerve terminals innervating salivary took maximally deep and prolonged inspi-
glands? rations and expirations. Thereafter he
felt slight dizziness. What is the probable
A. Noradrenaline cause of this phenomenon?
B. Acetylcholine
C. Serotonin A. Drop of pCO2 in blood
D. Histamine B. Rise of pCO2 in blood
E. GABA C. Bronchi constriction
D. Arterial pressure rise
179. What mediator provides informati- E. Drop of pO2 in blood
on transmission from nerve terminations
of motoneurons to the fibers of skeletal 185. A child has got a burn. Burnt skin is
muscles? hyperemic, there are small vesicles full of
transparent fluid. What type of fluid is it?
A. Acetylcholine
B. Adrenaline A. Serous exudate
C. Noradrenaline B. Hemorrhagic exudate
D. Serotonin C. Purulent exudate
E. GABA D. Transsudate
E. Putrid exudate
180. Bacca fruit is typical for the following
representative of Solanaceae family: 186. As a result of reaction of mercury
excess with diluted nitric acid the followi-
A. Atropa belladonna ng gas will escape:
B. Hyoscyamus niger
C. Datura stramonium A. NO
D. Nicotiana tabacum B. NH3
E. Datura innoxia C. N2
D. N2 O
181. A male patient has pain in the right E. -
subcostal area, acholic feces. Decolourati-
on of feces is caused by deficiency of: 187. A biological system (living organism)
exchanges material and energy with the
A. Stercobilin environment. What system does it relate
B. Hemoglobin to?
C. Bilirubin
D. Bile acids A. Open, heterogenous
E. Skatole B. Isolated, heterogenous
C. Closed, homogenous
182. A patient is 50 years old. Ad a result D. Closed, heterogenous
of continuous improper feeding he got E. Open, homogenous
hypovitaminosis C. Lesion of connecti-
ve tissue is caused by low activity of the 188. A patient has been suffering from di-
following enzyme: abetes mellitus for 10 years. He was deli-
38
krok123.in.ua
Krok 1 Pharmacy 2008 20
40
krok123.in.ua
Krok 1 Pharmacy 2009 1
preparation is the coenzymatic form of 18. Nitrogen (I) oxide (N2 O) is applied for
the following vitamin: inhalation narcosis. It is obtained by heati-
ng of:
A. B1
B. B6 A. NH4 NO3
C. B12 B. NH3
D. C C. Cu(NO3 )2
E. P D. NH4 OH
E. NaNO3
13. During starvation the normal rate of
glucose in blood is sustained due to the 19. Quantitative determination of
gluconeogenesis stimulation. Which of pharmaceutical substances can be done
the following substances can be used as by means of acidimetry. Its titrant is
a source for glucose synthesis? the secondary standard solution of
hydrochloric acid. According to which
A. Alanine compound the precise concentration of
B. Adenine hydrochloric acid can be determined?
C. Ammonia
D. Nicotinamide A. Sodium tetraborate
E. Urea B. Oxalic acid
C. Potassium dichromate
14. It is known that infectious type B D. Sodium thiosulfate
hepatitis is a systemic disease caused by E. Magnesium sulphate
the type B hepatitis virus and characteri-
zed by a predominant liver affection. 20. Analysis of a patient’s urine revealed
Choose from the below given list the increased concentration of the uric acid.
drugs for the etiotropic therapy of this The patient was prescribed allopurinol.
infection: What is the biochemical mechanism of its
action?
A. Acyclovir
B. Penicillin A. Xanthine oxidase inhibition
C. Tetracycline B. Cyclooxigenase activation
D. Sulfanilamides C. Desaminase inhibition
E. Fluoroquinolones D. Phosphorylase inhibition
E. Nucleosidase inhibition
15. Choose a non-salt-forming oxide from
the following compounds: 21. Physical exercise results in an increase
in thermogenesis due to an increase in
A. N2 O heat production in the following structure:
B. CuO
C. P2 O5 A. Skeletal muscles
D. SO3 B. Heart
E. Na2 O C. Lungs
D. Liver
16. What substance can act as both oxi- E. Brain
dant and reducer in oxidation-reduction
reactions? 22. Plant pathogenic microorganisms
relate to various groups. Which of them
A. SO2 causes diseases of medicinal plants most
B. SO3 often?
C. CO2
D. P bO2 A. Fungi
E. CrO3 B. Viruses
C. Bacteria
17. Oxidative deamination of biogenic D. Actinomycetes
amines in the tissues is catalyzed by the E. Micoplasma
following enzyme:
23. It is known that proteins, fats and
A. Monoaminooxidase carbohydrates are digested by means of
B. Aspartate transaminase proteases, lipases and amylases, respecti-
C. Alanine transaminase vely. Which of digestive juices contains all
D. Decarboxylase these groups of enzymes enough for di-
E. Acetylcholinesterase gestion?
42
krok123.in.ua
Krok 1 Pharmacy 2009 3
43
krok123.in.ua
Krok 1 Pharmacy 2009 4
A. H2 N − NH2 A. Hesperidium
B. N 3 B. Pepo
C. H2 N − CH3 C. Multicoccus
D. C6 H5 NH2 D. Drupe
E. C6 H5 NHNH2 E. Bacca
34. When computing quantities of 39. Enzymes (biological catalysts) are
adjuvant substances required to make li- used as pharmacologic preparations.
quid drug forms isotonic, the values of What is the mechanism of enzyme acti-
isotonic quotients are used. What is the on in the biochemical reactions?
quotient for zinc sulphate if known that
it dissociates completely in an aqueous A. They reduce the energy of reaction
solution? activation
B. They increase the energy of reaction
A. 2 activation
B. 0 C. They inhibit the reaction process
C. 1 D. They change the constant of the reacti-
D. 3 on rate
E. 4 E. They change the reaction order
35. After a stomach resection a pati- 40. After a solution had been heated wi-
ent presented with weakness, skin pallor, th (NH4 )2 S2 O8 in presence of AgNO3 , it
face puffiness, enlargement of liver and turned crimson. What ions were present
spleen. Analysis of the peripheral blood in the solution?
revealed megaloblasts and megalocytes;
hyperchromatism (colour index - 1,3). A. Mn2+
What type of anaemia is observed in this B. F e3+
patient? C. F e2+
D. Co2+
A. B12 -deficient E. Cu2+
B. Haemolytic
C. Hypoplastic 41. A solution under examination
D. Iron-deficient was added to the solution of F eSO4
E. Toxic in presence of concentrated H2 SO4 .
Generation of a brown ring indicates
36. Water-soluble vitamins take coenzyme presence of:
form in an organism. Thiamine di-
phosphate is coenzyme of the following A. Nitrate ions
vitamin: B. Acetate ions
C. Carbonate ions
A. B1 D. Oxalate ions
B. B2 E. Phosphate ions
C. C
D. B6 42. Specify standard substances used
E. B12 for standardization of titrant soluti-
ons (NaOH, KOH) in the alkalimetric
37. Quantitative determination of calcium method:
chloride is carried out by method of direct
chelatometric titration. Choose an indi- A. Oxalic and succinic acids
cator for fixation of the titration endpoint: B. Acetic and succinic acids
C. Formic and acetic acids
A. Eriochrome black T D. Sulphanilic and oxalic acids
B. Phenolphthalein E. Sulphanilic and salicylic acids
C. Methyl red
D. Eosin 43. Irritation of the sympathetic nerve
E. Starch in an experimental dog induces quanti-
tative and qualitative alterations in the
38. One of the examined soft fruits is saliva composition. What alterations are
characterized by essential-oil exocarp, induced?
spongioid mesocarp and overgrown
endocarp that consists of juice saccules.
What fruit was under examination?
44
krok123.in.ua
Krok 1 Pharmacy 2009 5
E.
46
krok123.in.ua
Krok 1 Pharmacy 2009 7
vision:
A. Bryophyta
B. Lycopsida
C. Equisetophyta
D. Pteridophyta
E. Gymnospermae
A.
62. A fruit under examination is
pseudomonocarpic, with woody pericarp
and one seed. The seed cuticle remains
unfused with the pericarp. Such fruit is
called:
A. Nut
B. B. Cremocarp
C. Achenocarp
C. H2 NOC − CH2 − CH2 − CONH2 D. Caryopsis
D. H2 NOC − CH2 − CH2 − CH2 − CONH2 E. Pseudomonocarpic drupe
63. The birch has compound inflorescences
with drooping main axis bearing dichasia
composed of unisexual cells. Therefore,
this inflorescence is called:
E. A. Ament
B. Raceme
C. Spadix
59. Ammonia is generated in different ti- D. Spike
ssues and organs and then transported to E. Glomus
liver for detoxication and conversion into
urea. What amino acid transports it from 64. A female patient bitten by a stray dog
skeletal muscles to liver? came to a surgery. Wide lacerated wounds
were localized on the patient’s face. What
A. Alanine treatment-and prevention aid should be
B. Histidine rendered in order to prevent rabies?
C. Glycin
D. Serine A. Immunization with the antirabic vacci-
E. Valine ne
B. Combined antibiotic therapy
60. For production of phenol ether it is C. Hospitalization, injection of diphtheria-
necessary to cause reaction of sodium pertussis-tetanus vaccine
phenoxide with: D. Hospitalization, medical surveillance
E. Urgent injection of normal gamma-
globulin
65. It is suspected that the workers
of a serum drugs plant at a regional
hemotransfusion station are carriers of
pathogenic staphylococcus aureus. In
order to detect staphylococcus carriage,
the material from the nasopharynx of the
A. CH3 Cl workers should be inoculated into the
B. CH3 OH following medium:
C. CH4
D. CH3 NH2 A. Egg-yolk-salt agar
E. CH3 C ≡ N B. Endo agar
C. Meat infusion broth
61. A higher nonvascular plant has di- D. Kessler medium
stinct alternation of dominant sexual E. Blood agar
(gametophyte) and reduced asexual
(sporophyte) generations. This indicates 66. Analysis of sputum obtained from
that the plant belongs to the following di- a patient with suspected pneumonia
47
krok123.in.ua
Krok 1 Pharmacy 2009 8
48
krok123.in.ua
Krok 1 Pharmacy 2009 9
A. Saturated calomel
B. Quinhydrone
C. Glass
A. D. Hydrogen
E. Zinc
79. Sodium nitrite is used in medicine as
a vasodilating drug against stenocardia.
NaNO2 acts as reducer with the following
B. compound:
A. KMnO4
B. H2 S
C. NH3
C. D. KI
E. NaHCO3
80. Most technological processed in
pharmaceutics run in heterogenous
D. systems. How many phases has an eutectic
composition under eutectic temperature
of two-component system?
A. 3
B. 2
C. 5
E. D. 4
E. 1
76. During the quantitative analysis carri- 81. Sulphur (IV) oxide is a constituent
ed out under the primary conditions, part of one of the most harmful envi-
a specific reagent to F e3+ cations is ronment pollutants called toxic smog.
K4 [F e(CN)6 ]. Their interaction gives a When dissolved in water, sulphur (IV) oxi-
precipitate of the following colour: de forms the following acid:
A. Blue A. Sulphurous
B. White B. Sulphuric
C. Brown C. Hydrosulphuric
D. Red D. Thiosulfate
E. Black E. Tetrathionate
77. From a patient with the symptoms 82. Micelle solutions of surfactants are
of acute meningitis the spinal fluid was applied in pharmaceutical production as
taken. Its smears contained gram-negative stabilizers and solubilizers. What soluti-
diplococci within the leukocytes and on of colloidal surfactants will have the
outside them. Which microorganism is the greatest value of critical concentration of
most likely cause of the disease? micelle formation?
A. Neisseria meningitidis A. C9 H19 SO3 Na
B. Haemophilus influenzae B. C14 H29 SO3 Na
C. Streptococcus pneumoniae C. C16 H33 SO3 Na
D. Candida albicans D. C12 H25 SO3 Na
E. Escherichia coli E. C10 H21 SO3 Na
78. Potentiometric method of pH 83. In the pharmaceutical industry, the
determination is regarded as the most micelle-forming solutions of surface-
universal and enters into the Nati- active substances are used for producti-
onal Pharmacopeia of Ukraine. Which on of water-soluble preparations out of
electrode is used as a reference electrode? water-insoluble substances, for example
vitamins A and E. The critical concentrati-
on of micelle formation has the lowest
value in the solutions of the following
substances:
49
krok123.in.ua
Krok 1 Pharmacy 2009 10
A. Calcium chloride
B. Anhydrous calcium oxalate
C. Monohydrous calcium oxalate
A. D. Calcium carbonate
E. Calcium hydroxide
107. A man’s tip of tongue was processed
with an anesthetic solution. Therefore he
B. will loose the sense of the following taste:
A. Sweet
B. Bitter
C. Sour
D. Salty
E. Bitter and salty
A.
A.
B. B.
C.
C.
D.
D.
E.
56
krok123.in.ua
Krok 1 Pharmacy 2009 17
149. The best swelling of gelatine will be 154. As a result of staining of a plant mi-
57
krok123.in.ua
Krok 1 Pharmacy 2009 18
A. Transamination
A. Atropa belladonna B. Reduction-oxidation
B. Hyoscyamus niger C. Phosphorilation
C. Datura stramonium D. Methylation
D. Nicotiana tabacum E. Hydrolysis
E. Datura innoxia
178. Before diving experienced divers first
173. A patient was found to have an take several deep breaths. They do it in
increase in total bilirubin concentration order to:
in plasma at the expense of indirect bi-
lirubin; high rate of stercobilin in feces A. Remove as much as possible CO2
and urine; normal rate of direct bilirubin. B. Reduce functional residual capacity of
What jaundice is it? lungs
C. Increase lung vital capacity (LVC)
A. Haemolytic D. Increase total lung capacity (TLC)
B. Mechanic E. Increase respiratory volume (RV)
C. Gilbert’s syndrome
D. Parenchymatous 179. As a result of reaction of mercury
E. Physiological excess with diluted nitric acid the followi-
ng gas will escape:
174. A patient has been administered
L-carnitine. This preparation ensures A. NO
transmembrane transfer of the following B. NH3
substances: C. N2
D. N2 O
A. Higher fatty acids E. -
B. Amino acids
C. Purine nucleotides 180. A 47-year-old patient with an arm
D. Pyrimidine nucleotides injury was delivered to a hospital in pain
E. Glucose shock condition. Objectively: the pati-
ent is in grave condition, with mental
175. Coordination number of iron in confusion; integuments are moist, pale,
the potassium hexacyanoferrate (II) acrocyanotic. There are also tachypnea,
K4 [F e(CN)6 ] is: fall in the arterial pressure, tachycardia.
A. 6 What type of hypoxia is prevailing in this
B. 2 patient?
C. 4 A. Circulatory
D. 3 B. Haemic
E. 8 C. Tissue
176. A patient is 50 years old. Ad a result D. Respiratory
of continuous improper feeding he got E. Substrate
hypovitaminosis C. Lesion of connecti- 181. A patient suffering from the essenti-
ve tissue is caused by low activity of the al hypertension presents with an increase
following enzyme: in the arterial pressure up to 180/110 mm
A. Proline hydroxylase Hg; dyspnea, cyanosis, tachycardia; heart
B. Alanine aminotransferase borders are dilated to the left, in lungs
C. Pyruvate carboxylase moist rales are present. What signs of
D. Tryptophane hydroxylase urgent compensation for cardiac failure
E. Glutaminase are observed?
into a coma, there appeared noisy deep line with ordinate axis of the equilibrium
breathing. Deep inspirations took turns diagram of a two-component system:
with forced expirations with assistance of
expiratory muscles. What form of respi- A. С = 0
ration disorder is it? B. С = 2
C. С = 1
A. Kussmaul’s respiration D. С = –1
B. Stenotic respiration E. С = 3
C. Tachypnea
D. Cheyne-Stokes respiration 188. After taking phenacetin a pati-
E. Biot’s respiration ent complained about sore throat
and impossibility of deglutition. An
183. The product of potassium otolaryngologist made a dignosis of
permanganate reduction in the neutral necrotic angina. In blood: Hb- 130 g/l,
medium has the following chemical erythrocytes - 4, 5 · 1012 /l, leukocytes -
formula and colour: 3, 0 · 109 /l, among them lymphocytes -
75%, neutrophils - 10%, eosinophils - 5%,
A. MnO2 , brown monocytes - 10%. What type of white
B. MnO2 , green blood cell disorder is it?
C. K2 MnO4 , green
D. K2 MnO4 , violet A. Neutropenia
E. MnSO4 , colourless B. Neutrophilia
C. Monocytosis
184. What is maximal valency of ni- D. Eosinophylia
trogen in consideration of donor-acceptor E. Lymphopenia
mechanism of covalent bond?
189. 3 years ago a patient was diagnosed
A. 4 with chronic glomerulonephritis. The pati-
B. 1 ent has got multiple edemata within the
C. 2 last 6 months. What is the cause of their
D. 3 development?
E. 5
A. Proteinuria
185. Microscopy of a leaf epidermis B. Hyperaldosteronism
of Convallaria majalis showed that the C. Injection of non-steroidal anti-
stomata had four accessory cells. Two of inflammatory preparations
them were lateral, and two other were D. Glucocorticoid treatment
polar. What type of stomatal mechanism E. Vasopressin hyperproduction
is it?
190. One of the herbarium specimens of
A. Tetracytic medicinal plants relates to the Asteraceae
B. Diacytic family. This plant is:
C. Anisocytic
D. Anomocytic A. Arctica lappa
E. Paracytic B. Atropa belladonna
C. Cassia acutifolia
186. Medical examination of a dairymaid D. Urtica dioica
revealed affection of the locomotive E. Rubus idaeus
system, vision impairment, disorder of the
nervous and other systems. To confirm the 191. Examination of a medicinal herb
diagnosis the patient was referred for a revealed that its leaves were divided down
serological assay (Wright’s reaction) and to the base of the leaf blade with segments
Burnet’s skin allergy test. What was the radiating from a common point in a fan
provisional diagnosis? manner. These leaves are:
A. Brucellosis A. Palmatisected
B. Tularemia B. Pinnatisected
C. Anthrax C. Palmatipartite
D. Rheumatism D. Pinnatipartite
E. Leptospirosis E. Palmatilobate
187. Specify the number of degrees of 192. What is the primary standard for
freedom for intersection of the liquidus standardization of Hg2 (NO3 )2 solution?
61
krok123.in.ua
Krok 1 Pharmacy 2009 22
62
krok123.in.ua
Krok 1 Pharmacy 2010 1
A. Ag + HNO3 A. SO2
B. SO3
B. AgCl + NH4 NO3
C. CO2
C. Ag + KNO3
D. Ag2 O + KNO3 D. P bO2
E. CrO3
E. AgCl + NaNO3
20. Heparin is the direct-acting anti-
15. Solution of Trilon B is the ti-
trant in chelatometry. It makes complex coagulant that reduces blood coagulati-
on and prevents thrombosis. Its action is
compounds with metal cations irrespecti-
based upon the following phenomenon:
ve of their valency with the following
proportion: A. Protective power of colloids
A. 1:1 B. Syneresis
C. Thixotropy
B. 1:3
C. 1:2 D. Micelle formation
E. Dialysis
D. 2:1
E. 3:1 21. Examination of a patient revealed
an increase in low-density lipoprotein
16. High-grade deficit of the ascorbic acid
causes development of scorbutus. This concentration in blood serum. The pati-
ent can be expected to have the following
pathology develops due to the disturbed
synthesis of the following connective ti- disease:
ssue protein:
64
krok123.in.ua
Krok 1 Pharmacy 2010 3
A. Atherosclerosis A. Paraplegia
B. Pneumonia B. Tetraplegia
C. Glomerulonephritis C. Monoplegia
D. Acute pancreatitis D. Hemiplegia
E. Gastritis E. Paraparesis
22. Nitrogen (I) oxide (N2 O) is applied for 28. In course of an experiment the experi-
inhalation narcosis. It is obtained by heati- menters are stimulating a sympathetic
ng of: nerve responsible for heart innervation.
What changes in cardiac activity can be
A. NH4 NO3 expected?
B. NH3
C. Cu(NO3 )2 A. Increase in heart rate and force
D. NH4 OH B. Decrease in heart force
E. NaNO3 C. Decrease in heart rate
D. Deceleration of excitement conduction
23. After examination a patient has E. Increase in heart rate
been diagnosed with alkaptonuria. This
pathology is caused by the deficit of the 29. Sodium arsenate solution can be di-
following enzyme: stinguished from the arsenite solution by
means of the following reagent:
A. Homogentisic acid oxidase
B. Diamine oxidase A. Magnesia mixture
C. Acetylcholinesterase B. Potassium sulphate
D. Thyroxin hydroxylase C. Potassium nitrate
E. Monoamine oxidase D. Sodium chloride
E. Sodium fluoride
24. Plant pathogenic microorganisms
relate to various groups. Which of them 30. Passive and active transport of
causes diseases of medicinal plants most substances is realized through the cell
often? membrane. Name the type of active
transport by which the membrane changes
A. Fungi its structure:
B. Viruses
C. Bacteria A. Endocytosis
D. Actinomycetes B. Osmosis
E. Micoplasma C. Filtration
D. Diffusion
25. Calcium hydrogen sulphide E. Facilitated diffusion
hexahydrate is often used in the
cosmetological practice. Specify the the 31. Disperse systems are widely used in
formula of his salt: the pharmaceutical practice. The evidence
of colloidal state is the passing of light
A. Ca(HS)2 · 6H2 O through the system. In this case the beam
B. CaS · 6H2 O of light:
C. CaSO3 · 6H2 O
D. Ca(HSO3)2 · 6H2 O A. Is diffused in form of light cone
E. CaSO4 · 6H2 O B. Is reflected
C. Is adsorbed
26. Iodometric determination of D. Is refracted
formaldehyde in formaline can be done E. Penetrates into the particle
by the back titration. Iodine surplus is ti-
trated with the standard solution of: 32. Qualitative determination of the
following compound is accompanied by
A. Sodium thiosulphate blue stain of the ether layer:
B. Sodium nitrate
C. Sodium sulphate A. H2 O2
D. Sodium carbonate B. Cl2
E. Sodium phosphate C. Na2 HP O4
D. MnSO4
27. As a result of spine injury a female E. F eSO4
patient has no voluntary movements of
her lower limbs. This disorder is called: 33. Corolla of the origanum flower is
zygomorphic, sympetalous and consists of
65
krok123.in.ua
Krok 1 Pharmacy 2010 4
a tube and two limbs. The upper limb is in presence of concentrated H2 SO4 .
bilobate and the lower is trilobate. Such Formation of a brown ring indicates
corolla is called: presence of:
A. Bilabiate A. Nitrate ions
B. Unilabiate B. Acetate ions
C. Lingulate C. Carbonate ions
D. Thimble-like D. Oxalate ions
E. - E. Phosphate ions
34. Water-soluble vitamins take coenzyme 39. A patient complains of pain behind
form in an organism. Thiamine di- the breastbone on the left, perspiration
phosphate is the coenzyme of the followi- and palpitation. Which of the followi-
ng vitamin: ng enzymes should be found in blood
in order to confirm the diagnosis of
A. B1 myocardium infarction?
B. B2
C. C A. AspAT, CPK, LDH-1
D. B6 B. AlAT, aldolase, LDH-4
E. B12 C. Amylase, alkaline phosphatase, AlAT
D. Acid phosphatase, LDH-5, LDH-4
35. Biochemical function of water-soluble E. α-fetoprotein, aldolase, CPK
vitamins depends on their ability to turn
into the coenzymatic forms. Specify the 40. For tuberculosis prevention the
coenzymatic form of the vitamin B2 (ri- newborns got an injection of a vaccine.
boflavin): What vaccine was used?
67
krok123.in.ua
Krok 1 Pharmacy 2010 6
A. Hydroxylation
B. Oxidation
C. Methylation
D. Acetylation
E. Reduction
57. For production of phenol ether it is
necessary to cause reaction of sodium
phenoxide with:
A.
B.
C.
A. CH3 Cl
D. B. CH3 OH
C. CH4
D. CH3 NH2
E. CH3 C ≡ N
58. Polysaccharide cellulose consists of the
E. remains of the following monosaccharide:
69
krok123.in.ua
Krok 1 Pharmacy 2010 8
70
krok123.in.ua
Krok 1 Pharmacy 2010 9
A. Mg 2+
B. F e3+ A. Glycol
C. Mn2+ B. Ketone
D. F e2+ C. Carboxylic acid
E. Ni2+ D. Aldehyde
E. Epoxide
83. A flower has the androecium consisti-
ng of two long and two short stamens. 89. Study of the antibioticogram of the
Therefore the flower’s androecium is: pure salmonella culture revealed multiple
antibiotic resistance. What factor might
A. Didynamous have caused this effect?
B. Tetradynamous
C. Diadelphous A. R-plasmids
D. Tetradelphous B. Chromosomal mutations
E. Polyadelphous C. F-plasmids
D. Temperate phages
84. Potassium dichromate K2 Cr2 O7 is E. Transposons
applied as oxidant in acidic medium. What
is the product of reduction of dichromate- 90. After a 5-year-old child has been
ion Cr2 O72− under these conditions? brought home from the kindergarten he
presented with weakness, headache, body
A. Cr3+ temperature rise up to 37, 5o C. What peri-
B. Cr(OH)3 od of disease develompent is the case?
C. Cr(OH)2
A. Prodromal
D. [Cr(OH)6]3− B. Latent
E. Cr2 O3 C. Incubative
85. In course of an experiment a dog has D. Recovery
been injected a preparation that reduces E. Fastigium
secretory and motor activity of stomach. 91. It is required to determine the amount
What preparation is it? of sodium salicylate in a solution. What ti-
A. Atropine trimetric method can be applied for the
B. Histamine quantitative determination of aromatic
C. Secretin compounds?
D. Acetylcholine A. Bromometry
E. Gastrin B. Mercurimetry
86. What ion mechanism is responsible for C. Cerimetry
the development of depolarization phase D. Argentometry
of action potential? E. Chelatometry
A. Sodium influx into the cell 92. When copper (II) hydroxide enters
B. Sodium outflux into reaction with alkali, complex
C. Potassium influx into the cell compounds are produced. What is the
D. Potassium outflux coordination number of copper in such
E. Calcium influx into the cell compounds?
A. 2,4,6-tribromaniline A. o-xylol
B. 2,4-dibromaniline B. Salicylic acid
C. 2,6-dibromaniline C. 1,2-dichlorobenzene
D. 2-bromaniline D. 2-chlorobenzoic acid
E. 4-bromaniline E. m-xylol
94. For treatment of the psychosis a pati- 99. Interaction of lactic acid with SOCl2
ent was administered the neuroleptic excess will result in production of the
aminazine. The main way of its bi- following compound:
otransformation in the organism is
induction of microsomal oxidation. Speci-
fy the principal component of this system:
A. Cytochrome R-450
B. Cytochrome C
C. Cytochrome oxidase
D. NAD-dehydrogenase
E. CoQ-reductase
95. Every year during the plant blossomi-
ng a female patient develops acute
catarrhal inflammation of conjuncti- A.
va and nasal mucosa that is the cli-
nical presentation of an allergy. These
symptoms relate to the following type of
allergic reactions:
A. Anaphylactic
B. Cytotoxic B.
C. Immune complex
D. Cell-mediated
E. Cellular dysfunction
96. A patient is suspected to have the C.
typhoid fever. What method of laboratory
diagnostics would be the most appropri-
ate for confirmation of this diagnosis in
the first week of disease?
A. Hemoculture identification
B. Urine culture identification D.
C. Myeloculture identification
D. Biliculture identification
E. Coproculture identification
97. Inoculation of hen’s embryos is the
main method of detection of influenza E.
virus. In order to neutralize associated
bacterial flora in the material under
examination (nasopharyngeal lavage) it 100. A group of alpinists climbing to
is necessary to add beforehand: the top had their blood tested. The test
revealed erythrocytosis and increase in
A. Antibiotics hemoglobin rate. What type of hypoxia
B. Eubiotics caused the stimulation of erythropoiesis
C. Fluorescent serum in the bone marrow?
D. Leukocytic interferon
E. Ant-influenza gamma globulin A. Hypoxic
B. Combined
98. Choose the initial compound for one- C. Hemic
stage synthesis of phthalic acid: D. Circulatory
E. Tissue
101. A 42-year-old patient suffering from
72
krok123.in.ua
Krok 1 Pharmacy 2010 11
73
krok123.in.ua
Krok 1 Pharmacy 2010 12
A. Cholesterol
B. Albumine
C. Haemoglobin
D. Urates
A. E. Oxalates
115. What method of titrimetric analysis
can be applied for the quantitative
B. determination of sulphuric acid by means
of the potassium hydroxide solution?
A. Alkalimetry
B. Acidimetry
C. C. Oxidation-reduction
D. Precipitation
E. Complexation
116. Ions of which chemical element have
an impact on the electrolytic balance of
D. cerebral tissues. What salt of this element
is used for treating the psychic disorders?
A. Li, Li2 CO3
B. Cl, NaCl
C. I, KI
E. D. Ca, CaCl2
E. Mg, MgSO4
112. In order to choose an indicator duri- 117. Drug production commonly involves
ng the acid-bace titration a titration curve the processes of adsorption and ion
is made which is the dependence of: exchange. What ion is selectively
adsorbed from the aqueous solution based
A. pH solution from the volume of the on silver chloride crystal?
added titrant
B. pH solution from the concentration of A. Ag +
the added titrant B. H +
C. pH solution from the volume of the C. NO 3
solution under analysis D. Cu2+
D. Concentration of the solution under E. OH
analysis from pH solution
E. pH solution from the temperature 118. Osmotic pressure is an important
characteristic of biologic fluids. Osmotic
113. Nephron is the structural and functi- pressure variates with time in the followi-
onal unit of the kidneys. The process of ng solution:
filtration takes place in the following part
of it: A. Silver chloride sol
B. Glucose
A. Bowman’s capsule C. Calcium sulphate
B. Henle’s loop D. Sodium chloride
C. Collecting tubule E. Magnesium sulphate
D. Proximal tubule
E. Distal tubule 119. 3-aminopropane acid is included in
pantothenic acid which is a component of
114. Inflammatory processes in the gall coenzyme A. What reaction takes place in
bladder exert negative influence on the course of heating of this acid?
colloidal properties of bile. This may lead
to gallstone formation. One of the causes A. Elimination (detachment)
of their formation is the crystallization of B. Substitution
the following substance: C. Addition
D. Rearrangement
E. Reduction
120. The given reaction is called:
74
krok123.in.ua
Krok 1 Pharmacy 2010 13
A. HCl, NaOH
B. HCl, H2 SO4
C. KOH, NaOH
A. D. NaOH, KCl
E. H2 SO4, K2 SO4
146. Nitritometric determination of
compounds containing primary aromatic
B. amino group can be carried out under the
following conditions:
A. With observation of all the mentioned
conditions
C. B. At a temperature up to 10o C
C. With adding of the crystalline KBr
(catalyst)
D. Chloric acid excess
E. Slow titration
D.
147. Under anaerobic conditions duri-
ng glycolysis ATP is synthesized by
the way of substrate phosphorylation.
E. This process uses energy of other high-
energy compounds. Specify one of such
compounds:
142. What is the osmotic pressure of medi-
cinal solutions used as blood isotonics? A. Phosphoenol pyruvate
B. Glucose 6-phosphate
A. 740 - 780 kPa C. Lactate
B. 420 - 448 kPa D. Pyruvate
C. 900 - 960 kPa E. Glucose
D. 600 - 670 kPa
E. 690 - 720 kPa 148. A patient with acute pneumonia has
an edema and hardening of pulmonary ti-
143. Under what conditions the limited ssue. What cells are the first to infiltrate
swelling of gelatine turns into the unlimi- the inflammation zone and provide the
ted one? effective protection from the bacterial
infection?
A. Heating
B. Cooling A. Neutrophils
C. In presence of P O43− ions B. Monocytes
C. Thrombocytes
D. In presence of Cl− ions D. Eosinophils
E. In presence of H + ions whose E. Basophils
concentration is equal to their concentrati-
on in the isoelectric point 149. Heart automatism is possible due
to the atypical cardiomyocytes formi-
144. A 57-year-old worker at an asphalt ng the cardiac conduction system. What
plant complains of weakness, cough wi- part of this system is the primary cardiac
th blood-streaked sputum, chest pain. pacemaker?
He has been diagnosed with lung cancer.
What is the first stage of carcinogenesis? A. Sinoatrial node
B. Purkinje’s fibers
A. Transformation C. Atrioventricular node
B. Promotion D. His’ bundle
C. Activization E. His’ bundle branches
D. Progression
E. Induction 150. Examination of a patient revealed
reddening of oral mucosa, cracks on
145. Choose a pair of titrants for the quali- the lips and mouth corners, face skin
tative determination of ammonia in a dryness and desquamation, conjunctiva
solution by the method of back titration: inflammation, vasculature invasion into
the cornea. The possible cause of this
77
krok123.in.ua
Krok 1 Pharmacy 2010 16
pathology is the deficit of the following of bile inflow to the duodenum. This wi-
vitamin: ll cause the failure of hydrolysis of the
following substances:
A. B2
B. C A. Fats
C. E B. Carbohydrates
D. K C. Proteins
E. D D. Fats and carbohydtares
E. Proteins and carbohydrates
151. During the practical training the
students placed the isolated frog’s heart 156. As a result of sulfonation of
into a solution. This caused the cardiac naphthalene with concentrated sulfuric
arrest in diastole. What solution was the acid at a temperature over 160o C the
heart placed into? following substance is produced:
A. 3% solution of KCl A. 2-naphthalensulfonic acid
B. 1% solution of NaCl B. 1-naphthalensulfonic acid
C. 3% solution of NaCl C. 3-naphthalensulfonic acid
D. 1% solution of CaCl2 D. 4-naphthalensulfonic acid
E. 0,1% solution of MgCl2 E. 5-naphthalensulfonic acid
152. Which atoms of carbon in the given 157. One of the plants under examinati-
compound on has a zygomorphic flower and papili-
onaceous corolla. This plant is called:
A. Melilotus officinalis
B. Mentha piperita
C. Valeriana officinalis
are in the second valence state (sp2 - D. Urtica dioica
hybridization)? E. Rosa canina
82
krok123.in.ua
Krok 1 Pharmacy 2011 1
A. 1000 bacteria and 100 mold fungi 21. Electrolyte solutions are medicinal
B. 500 bacteria and 50 mold fungi preparations. What is the maximum value
C. 250 bacteria and 25 mold fungi of isotonic coefficient for MgSO4 soluti-
D. 500 bacteria and 200 mold fungi on?
E. 1500 bacteria and 150 mold fungi A. 2
16. Antibiotics can be classified accordi- B. 4
ng to various principles. According to the C. 3
action mechanism cephalosporins relate D. 5
to the following group: E. 7
23. Corolla of the origanum flower is What is the mechanism of enzyme acti-
zygomorphic, sympetalous and consists of on in the biochemical reactions?
a tube and two limbs. The upper limb is
bilobate and the lower is trilobate. Such A. They reduce the energy of reaction
corolla is called: activation
B. They increase the energy of reaction
A. Bilabiate activation
B. Unilabiate C. They inhibit the reaction process
C. Lingulate D. They change the constant of the reacti-
D. Thimble-like on rate
E. - E. They change the reaction order
24. Structure of proteins includes protei- 29. Depressurization of the cabin at an
nogenic amino acids. What is the positi- altitude of 19 km led to instantaneous
on of the amino group in the structure of death of pilots. What is its cause?
these amino acids?
A. Explosive decompression
A. α-position B. Hematencephalon
B. β-position C. Myocardial infarction
C. γ-position D. Bleeding
D. δ-position E. Respiratory centre paralysis
E. -position
30. After a girl had accidentally eaten
25. Biochemical function of water-soluble inedible mushrooms she was admitted
vitamins depends on their ability to turn to the resuscitation unit with symptoms
into the coenzymatic forms. Specify the of impaired consciousness, arterial
coenzymatic form of the vitamin B2 (ri- hypotension, anuria, hyperazotemia.
boflavin): What kind of renal dysfunction is it?
A. Х = OH
B. Х = COOH
C. Х = NO2 A. Tetrachloromethane
D. Х = CHO B. Chloroform
E. Х = SO3 H C. Chloroethanol
D. Ethane
36. Gastric juice of a patient has decreased E. Chloromethane
concentration of enzymes. What secretory
cells of stomach display disfunction? 41. 1M sulphuric acid solution was added
to the solution under study. This resulted
A. Chief cells of glands in formation of white sediment that was
B. Parietal cells of glands soluble in the alkalies. This indicated that
C. Gland mucocytes the solution contains:
D. Cells of tegumental epithelium
E. G-cells A. Plumbum cations
B. Calcium cations
37. Yield of medical products can C. Barium cations
be enhanced by proper choice of D. Argentum cations
temperature conditions during their E. Mercury (I) cations
production. What equation determines
dependence of equilibrium constant from 42. A solution contains cations of zinc
the temperature under constant pressure? and aluminum. Specify the reagent that
enables to detect cations of zinc in this
A. Isobaric lines of chemical reaction solution:
B. Isotherms of chemical reaction
C. Kirchhoff equation
D. Isochores of chemical reaction
E. Gibbs-Helmholtz equation
38. During identification of a perennial
herb of Ranunculaceaе family it was found
to have: apical flowers of regular form
86
krok123.in.ua
Krok 1 Pharmacy 2011 5
87
krok123.in.ua
Krok 1 Pharmacy 2011 6
75. As a result of the reaction of cyclic 79. Chlorophyll, the green pigment of
plants, is a chelate compound. Specify the
90
krok123.in.ua
Krok 1 Pharmacy 2011 9
91
krok123.in.ua
Krok 1 Pharmacy 2011 10
A. K2 Cr2 O7
B. NaCl
C. N2 B4 O7
D. K2 CO3
E. As2 O3
89. It is required to determine the amount
of sodium salicylate in a solution. What ti- A.
trimetric method can be applied for the
quantitative determination of aromatic
compounds?
A. Bromometry
B. Mercurimetry
C. Cerimetry
D. Argentometry B.
E. Chelatometry
90. A 56 year-old patient complains about
limitation of movements and pain in
hand joints, mainly at night. Objectively:
there is a disfiguring painful swelling of
affected joints. Blood and urine have high C.
concentration of uric acid. What disease
has developed?
A. Gout
B. Pellagra
C. Phenylketonuria
D. Alkaptonuria D.
E. Tyrosinosis
91. What elements of IIB group exhibit
amphoteric properties?
A. Zinc only
B. Zinc and cadmium
C. Cadmium and mercury
D. All elements E.
E. Mercury only
92. Interaction of aniline with of bromine 93. Analytical indication of effect of
water resulted of white precipitate. What potassium iodide solution upon unstai-
substance was produced? ned oxidizing anions in presence of
chloroform is:
A. Brown stain of free iodine
B. Settling down of white deposition
C. Change of aggregate state
D. Emission of gas bubbles
E. Origination of deposition and its soluti-
on in reagent excess
94. Every year during the plant blossomi-
ng a female patient develops acute
catarrhal inflammation of conjuncti-
va and nasal mucosa that is the cli-
nical presentation of an allergy. These
symptoms relate to the following type of
allergic reactions:
92
krok123.in.ua
Krok 1 Pharmacy 2011 11
A. Anaphylactic
B. Cytotoxic
C. Immune complex
D. Cell-mediated
E. Cellular dysfunction
95. A female patient consulted a doctor
about leg pain that arises usually toward
the evening; feet and shins edemata.
Objectively: leg skin is cyanotic, cold to A. Oxidation with potassium
the touch. What type of peripheral ci- permanganate
rculation disorder does the patient present B. Heating with sulphuric acid
with? C. Hydrogen peroxide action at a room
temperature
A. Venous hyperaemia D. Sodium hydroxide action at a room
B. Arterial hyperaemia temperature
C. Ischaemia E. Boiling in the open air
D. Stasis
E. Thrombosis 100. Which of the following compounds
forms a propionic aldehyde as a result of
96. Presence of the pathogenic mi- alkaline hydrolysis (H2 O, OH − )?
croorganisms in the air can be prognosti-
cated according to the content of sanitary-
indicative bacteria. Which bacteria indi-
cate immediate epidemiologic danger?
A. Haemolytic streptococci A.
B. Sarcinae
C. Mold fungi
D. Yeast fungi
E. Micrococci B.
97. A patient with low immunity, frequent
colds is recommended to take ascoruti-
ne as a more effective drug than ascorbic
acid. What constituent substance of this C.
preparation intensifies action of vitamin
C?
A. Vitamin P
B. Vitamin A
C. Glucose D.
D. Lactose
E. Vitamin D
98. Determination of ∆Boiling poi-
nt of water-alcohol mixtures is the
pharmacopoeial method of quantitative E.
determination of alcohol. Which method
enables to determine ∆Boiling point?
101. The 0,1 M solution of whi-
A. Ebullioscopy ch substance has the smallest ion
B. Cryoscopy concentration?
C. Enteroscopy
D. Osmometry A. CH3 COOH
E. Conductometry B. HCl
C. CaCl2
99. Toluol is converted to the benzoic acid D. H2 SO4
under the following conditions: E. NaNO3
102. Inflammatory processes in the gall
bladder exert negative influence on the
colloidal properties of bile. This may lead
to gallstone formation. One of the causes
93
krok123.in.ua
Krok 1 Pharmacy 2011 12
97
krok123.in.ua
Krok 1 Pharmacy 2011 16
D. Phenol + hydrogen
A. SE
B. AE
C. SR
D. SN
E. AN E. Phenol + chloromethane
146. A patient fell ill the day before, the
disease is acute with a predominance of
general toxic symptoms. With an account
for the epidemic situation in the city,
the doctor diagnosed the patient with
influenza A. What emergency etiotropic 149. Specify the electronic effects of the
treatment must be administered to this carboxyl group (−COOH) in a molecule
patient? of benzoic acid:
A. Rimantadine A. −I, −M
B. Oxolinic ointment B. −I
C. Gentamicin C. +I, −M
D. Inactivated influenza vaccine D. +I
E. Human gamma globulin E. −I, +M
98
krok123.in.ua
Krok 1 Pharmacy 2011 17
100
krok123.in.ua
Krok 1 Pharmacy 2011 19
been prescribed Linaetholum containi- with skin burns, impaired vision. He has
ng essential fatty acids. Which of the been diagnosed with albinism. It is caused
following acids is an obligatory part of by the deficiency of the following enzyme:
the preparation?
A. Tyrosinase
A. Linolenic B. DOPA-oxidase
B. Palmitic C. Phenylalanine hydroxylase
C. Crotonic D. Ornithine carbamoyl transferase
D. Stearic E. Arginase
E. Oleic
187. The properties in a series of oxides:
182. What cation of the 4th analytical Al2 O3 - SiO2 - P2 O5 - SO3 - Cl2 O7 change
group is present in a solution, if it is known in the following way:
that the reaction with a group reagent
causes formation of yellow precipitate? A. The properties increase from left to
right
A. Cr 3+ B. The properties decrease from left to
B. Zn2+ right
C. Sn2+ C. The properties first increase and then
D. Al3+ decrease
E. Sn(IV ) D. The properties do not change
E. The properties first decrease, then
183. A patient complains of general increase
weakness, dyspnea, palpitation. Exami-
nation revealed inflammation of the 188. Select the fruit that meets the descri-
mucous membrane of tongue, lips, especi- ption: monocarpic, dry, polyspermous, can
ally in the corners of mouth; inflammati- be split apart only in the ventral suture.
on and increased vascularization in the The seeds are located along the ventral
external membrane of eye. What is the li- suture:
kely cause of this pathological condition:
A. Follicle
A. Hypovitaminosis B2 B. Coccus
B. Hypovitaminosis A C. Fleshy stone fruit
C. Hypovitaminosis C D. Dry stone fruit
D. Hypervitaminosis A E. Follicetum
E. Hypervitaminosis B1
189. A patient with current coronary heart
184. Examination of a patient revealed an disease who had had two myocardial
increase in 17-ketosteroid concentration in infarctions of left ventricular wall presents
urine. Hydroxylation of 17-ketosteroids is with bubbling breathing and dyspnea.
possible with the enzymes of the following Pulmonary auscultation reveals moist
system: rales. What kind of heart failure is it?
A. Microsomal oxidation A. Left ventricular
B. Krebs cycle B. Right ventricular
C. Protein synthesis system C. Compensated
D. Pentose phosphate cycle D. Subcompensated
E. Ornithine cycle E. Combined
185. It is necessary to carry out preventive 190. A healthy 45-year-old man is sitti-
vaccination of a student group because ng in a chair reading a newspaper. What
of an occurrence of diphtheria. Which muscles ensure breathing in a sitting posi-
preparation should be used for the creati- tion?
on of the artificial active immunity?
A. Diaphragm and external intercostal
A. Diphtheria anatoxin muscles
B. Specific immunoglobulin B. Scalenes
C. DTP vaccine C. Diaphragm and internal intercostal
D. Inactivated bacteria vaccine muscles
E. Anti-diphtheria serum D. Internal intercostal muscles and straight
muscles of abdominal wall
186. A patient consulted a doctor about E. Sternocleidomastoid muscles
intolerance to the sun rays. He presents
102
krok123.in.ua
Krok 1 Pharmacy 2011 21
103
krok123.in.ua
Krok 1 Pharmacy 2012 1
1. Microscopic examination of
absorption zone of primary root cortex
revealed that it consisted mainly of
loose multilayer living parenchyma with
amyloid granules. It is called: A.
A. Mesoderm
B. Endoderm
C. Exoderm
D. Collenchyme B.
E. Phellogene
2. An excess of concentrated ammoni-
um hydroxide is a group reagent for the
cations of the VI analytical group (acid- C.
base classification) Co2+ , Ni2+ , Cd2+ ,
Cu2+ , Hg 2+ . In this case the following
substances are formed:
A. Water-soluble ammonia complexes D.
B. Hydroxides of acid-soluble cations
C. Stained, water-insoluble compounds
D. Hydroxides of alkali-soluble cations
E. Hydroxides of the cations insoluble in
the excess of ammonium hydroxide
E.
3. Both external and internal indicators
are used in the following titrimetric
method of analysis 5. Estimation of temperature of phase
transition at different pressures is of
A. Nitritometry great practical importance for modern
B. Alkalimetry pharmaceutical industry and can be done
C. Chelatometry by applying:
D. Permanganatometry
E. Argentometry A. Clapeyron-Clausius equation
B. Trouton’s rule
4. Which of the listed reactions indicates C. Gibbs’ phase rule
the basic properties of pyridine? D. Mendeleev-Clapeyron equation
E. Konovalov law
6. A patient with ischemic heart disease
has been administered inosine, which is an
intermediate metabolite in the synthesis
of:
A. Purine nucleotides
B. Metalloproteins
C. Lipoproteins
D. Glycoproteins
E. Ketone bodies
7. Examination of the lower limbs of a
40-year-old patient with coronary artery
disease and vascular disease of the
lower limbs (obliterating endarteritis)
revealed skin pallor and dystrophy, local
temperature decrease, sense shock, pain.
The patient is likely to have the following
disorder of the peripheral blood circulati-
on:
104
krok123.in.ua
Krok 1 Pharmacy 2012 2
105
krok123.in.ua
Krok 1 Pharmacy 2012 3
A. 2 A. pH 3,2-3,5
B. 4 B. pH 6,5
C. 3 C. pH 7,0
D. 5 D. pH 9,0
E. 7 E. pH 0,5-1,0
16. What substance blocks the conducti- 21. After taking phenacetin a pati-
on of excitation in the neuromuscular ent developed acute sore throat,
synapses? body temperature rise. Examination
allowed doctors to make a diagnosis
A. Curare of necrotic angina and agranulocytosis.
B. Noradrenaline Agranulocytosis can be characterized by
C. Adrenaline a decrease in the amount of the following
D. Somatostatin WBCs:
E. Aspartate
A. Neutrophils
17. The labels of some medications have B. Eosinophils
an inscription: Shake before use! This C. Basophils
warning is caused by: D. Lymphocytes
E. Monocytes
A. Sedimentation
B. Coagulation 22. A newborn born to an Rh-negative
C. Solubility of disperse systems mother (3rd pregnancy) presents with
D. Insolubility of disperse systems progressing jaundice, symptoms of CNS
E. None of the above excitation, anemia. What type of jaundice
is it?
18. Which of these formulas corresponds
with acetoacetic acid? A. Hemolytic
B. Parenchymatous
C. Obstructive
A. D. Parasitic
E. Toxic
23. A plant under examination has papili-
B. onaceous flower. This plant belongs in the
family:
A. Fabaceae
C. B. Scrofulariaceae
C. Ranunculaceae
D. Lamiaceae
E. Asteraceae
D.
24. Van’t Hoff’s rule is used for determini-
ng the shelf life of drugs. The temperature
coefficient of the rate of most chemical
E. reactions lies within the following range:
A. 2-4
19. Blood serum electrophoresis revealed B. 2-3
interferon. This protein is in the following C. 1-3
fraction: D. 3-4
E. 1-5
A. γ-globulins
B. α1 -globulins 25. In an emergency situation a scuba di-
C. α2 -globulins ver has quickly risen from the depths to
D. β-globulins the surface, which is against the rule. He
E. Albumins is unconscious, presents with respiratory
failure and cardiac activity disorder as a
20. Proteolytic enzymes of gastric juice result of decompression sickness. What
exhibit maximum activity in the medium complication may develop in the scuba di-
with the following pH: ver?
106
krok123.in.ua
Krok 1 Pharmacy 2012 4
110
krok123.in.ua
Krok 1 Pharmacy 2012 8
A. 3
B. 4
C. 2
D. 5 A.
E. 1
74. Cryoscopic constants of water,
benzene, chloroform, acetic acid and B.
camphor equal to 1,86; 5,12; 4,9; 3,9;
40,0 respectively. Which of these solvents
should be selected for the most accurate
determination of the molar mass of a
drug substance (nonelectrolyte) by the
cryoscopic method?
A. Camphor C.
B. Chloroform
C. Acetic acid
D. Benzene
E. Water
D.
75. Which of the listed carbonyl
compounds gives a positive iodoforme
reaction?
E.
A.
77. For the quantitative analysis of ethanol
the gas chromatography was used. Which
parameter was measured?
B. A. Peak height or area
B. Retention time
C. Retention volume
D. Peak width
E. Peak width at half height
C.
78. Potassium permanganate reacting with
hydrogen peroxide in acidic medium acts
as:
D. A. Oxidant
B. Reductant
C. Disproportionation agent
D. Oxidant and reductant
E. Does not act either as an oxidant, or as
E. a reductant
79. Specify the number of electrons
76. Which of these reactions can be used involved into formation of the isolated
to identify the primary amino group? conjugated system in the pyrimidine
molecule:
112
krok123.in.ua
Krok 1 Pharmacy 2012 10
of its conversion?
A. Pseudomonas
B. Proteus A. Alpha-glycerolophosphate
C. Clostridium B. Pyruvate
D. Shigella C. Lactate
E. Vibrio D. Choline
E. Acetyl coenzyme A
90. Microbiological analysis of medicinal
raw materials revealed capsular bacteria. 96. When ammonia enters into reacti-
What stain method was used to detect the on with acids, this results in formation
capsules? of ammonium salts. Which properties of
ammonia characterize this process?
A. Gin’s
B. Ziehl-Neelsen’s A. Ability to accept the hydrogen ions
C. Neisser’s B. Reductive properties
D. Gram’s C. Acidic properties
E. Ozheshko’s D. Oxidative properties
E. Ability to hydrolyze
91. The analyzed plant has hollow ribbed
stems, compound umbel inflorescence, 97. A patient had been diagnosed wi-
schizocarpic fruit (cremocarp) and is rich th right lung cancer and administered
in essential oils, which is a characteristic surgical treatment. After right-sided
of: pulmonectomy the patient presented wi-
th evident dyspnea. What form of respi-
A. Apiaceae ratory failure has developed in this pati-
B. Fabaceae ent?
C. Ericaceae
D. Brassisaceae A. Pulmonary restrictive
E. Asteraceae B. Central
C. Peripheral
92. Specify the colour of phenolphthalein D. Pulmonary obstructive
in the sodium sulfide solution: E. Thoracodiaphragmal
A. Crimson 98. What wave of ECG characterizes the
B. Colourless spread of excitation throughout the heart
C. Blue atria?
D. Yellow
E. Green A. P
B. R
93. In pharmaceutical technology an C. Q
important part is played by pressure, D. T
temperature, concentration. The reacti- E. S
on yield can be increased by lowering the
temperature of the following process: 99. Cardiac tones are the outer acoustic
manifestations of heart functioning. What
A. Exothermic is the cause of the II tone?
B. Endothermic
C. Isochoric A. Closure of the semilunar valves
D. Isobaric B. Closure of the cuspid valves
E. Adiabatic C. Vibration of the ventricle walls
D. Vibration of the atrium walls
94. To identify a drug by thin-layer E. Chest movements
chromatography the following parameter
is used: 100. In accordance with the requirements
of the pharmacopoeia, the non-sterile
A. Rf medicinal preparations may include mi-
B. n croorganisms. What micro-organisms
C. E, mV MUST NOT be present in them?
D. I, A
E. Kp
95. The intracellular metabolism of
glycerol starts with its activation. What
compound is formed in the first reaction
114
krok123.in.ua
Krok 1 Pharmacy 2012 12
A. Enterobacteria A. Plague
B. Ascomycetes B. Tuberculosis
C. Micrococci C. Leptospirosis
D. Mold fungi D. Brucellosis
E. Sarcinae E. Toxoplasmosis
101. While performing finger-nose test 106. Choose a name that corresponds to
the examinee could not touch the tip the formula: CH3 − C ≡ N:
of his nose with his fingertip having his
eyes closed. What structure of the central A. Acetic acid nitrile
nervous system is damaged? B. Acetamide
C. Acetic anhydryde
A. Cerebellum D. Acetoxime
B. Quadrigeminal plate E. Ethyl isocyanide
C. Cortex
D. Spinal cord 107. What data is required to determine
E. Thalamus the activation energy?
102. A patient has obstruction of the A. Constants of reaction rate at two
common bile duct. Which of these temperatures
substances is usually found in urine in B. Thermal energy of the reaction
such cases? C. Energy change of the system
D. Internal energy of the system
A. Bilirubin E. Reaction order
B. Ketone bodies
C. Uric acid 108. A patient with systemic lupus
D. Creatinine erythematosus has developed a diffuse
E. Glucose renal affection accompanied by protei-
nuria, hypoproteinemia, massive edema.
103. A continuous stay in the mountai- What is the mechanism of proteinuria
ns causes an increase of blood oxygen development in this case?
capacity. What is the possible reason for
this phenomenon? A. Autoimmune affection of glomeruli
B. Inflammation of renal tubules
A. Development of functional C. Ischemic affection of tubules
erythrocytosis D. Blood protein increase
B. Increase of P O2 rate in the air E. Affection of urinary tracts
C. Increase of P CO2 rate in the air
D. Decrease in respiratory rate and depth 109. What reactions are used in the
E. Development of gas acidosis methods of permanganatometry, di-
chromatometry, iodometry?
104. A student had to analyze an axi-
al plant organ characterized by radi- A. Oxidation-reduction
al symmetry, unlimited growth, positi- B. Precipitation
ve geotropism. It provided nutrition, C. Complexation
vegetative propagation, anchorage of D. Neutralization
plant in the soil. This organ was identi- E. Hydrolysis
fied as . . .
110. In order to bind hydrogen ions during
A. Root the identification of potassium ions with
B. Stem tartaric acid the following solution is used:
C. Leaf
D. Rhizome A. Sodium acetate
E. Seed B. Sodium hydroxide
C. Ammonia
105. A patient presents with fever, chi- D. Sulfuric acid
ll and cough. From his sputum the ovoid E. Hydrochloric acid
Gram-negative bipolar-stained bacilli wi-
th a delicate capsule were isolated. What 111. According to the Paneth-Fajans rule,
is the most likely diagnosis? the ion preferably adsorbed from a soluti-
on on the surface of a solid crystalline
adsorbent is the ion, which:
115
krok123.in.ua
Krok 1 Pharmacy 2012 13
116
krok123.in.ua
Krok 1 Pharmacy 2012 14
A. Reagin (anaphylactic)
A. Sensibilization B. Cytotoxic
B. Solubilization C. Immunocomplex
C. Mutual coagulation D. Cell-mediated
D. Colloidal protection E. Stimulating
E. Sol habituation
138. When determining the changes in
133. In order to increase the inhibitory membrane permeability during an acti-
processes in the CNS the pharmacological on potential it was established that duri-
agents are used that cause the following ng the depolarization phase the following
process on the postsynaptic membranes: movement predominates:
A. Hyperpolarization A. Movement of Na+ into the cell
B. Depolarization B. Movement of Na+ out of the cell
C. Afterdepolarization C. Movement of K + into the cell
D. Activation of sodium channels D. Movement of K + out of the cell
E. Activation of calcium channels E. Movement of Cl− into the cell
134. A 70-year-old patient has been found 139. Spore and pollen analysis revealed in
to have atherosclerosis of heart and the pollen some tetrahedral spores with a
brain vessels. Examination revealed the semi-circular base and a reticular surface,
changes in the lipid profile. Pathogenesis which may belong to:
of atherosclerosis is greatly influenced by
an increase in the following lipoproteins A. Lycopodiophyta
rate: B. Equisetiphyta
C. Bryophyta
A. Low-density lipoprotein D. Polypodiophyta
B. Very-low-density lipoproteins E. Pinophyta
C. Intermediate-density lipoproteins
D. High-density lipoprotein 140. In the practice of harvesting herbal
E. Chylomicrons raw material of Asteraceae family the term
"flowers"means both individual flowers
135. Calendula officinalis which a and inflorescences. However, the notion
representative of the aster fami- of "flowers"is botanically correct only for:
ly is characterized by the following
inflorescence type: A. Centaurea cyanus
B. Gnaphalium uliginosum
A. Flowerhead C. Arnica montana
B. Umbel D. Echinops ritro
C. Catkin E. Bidens tripartita
D. Glome
E. Cyme 141. What working solutions (titrants) are
used in the method of precipitation titrati-
136. Which medicinal plant of the on - Folgard method?
Asteraceae family has only disk flowers
in the flowerhead? A. AgNO3 and NH4 SCN
B. H2 SO4 and NaOH
A. Three-part beggarticks (Bidens triparti- C. Na2 S2 O3 and K(I3 )
ta) D. KMnO4 and KBrO3
B. Dandelion (Taraxacum officinale) E. HClO4 and KOH
C. Echinacea purpurea
D. Cornflower (Centaurea cyanus) 142. In which of these reactions hydrogen
E. Common yarrow (Achillea millefolium) acts as an oxidizing agent?
A. Basidiomycetes A. Bi(OH)3
B. Ascomycetes B. Sb(OH)3
C. Zygomycetes C. As(OH)3
D. Deuteromycetes D. H3 P O3
E. Oomycetes E. -
154. A laboratory received a sample of 159. What is the equivalent of Al(OH)3
water used in drug production for sanitary in the reaction Al(OH)3 + 2HCl =
and virological analysis. What group of vi- Al(OH)Cl2 + 2H2 O?
ruses will indicate faecal contamination of
water and thus the need for its additional A. 1/2 mol
purification? B. 1/3 mol
C. 1 mol
A. Picornaviridae D. 2 mol
B. Herpesviridae E. 3 mol
C. Orthomyxoviridae
D. Retroviridae 160. Food rich in carbohydrates at fi-
E. Flaviviridae rst increases the blood glucose and then
decreases its rate due to the insulin acti-
155. Specify the indicator of the protective on. What process is activated by this
properties of high-molecular compounds hormone?
of body that promote the keeping of calci-
um, phosphate and carbonate in blood A. Synthesis of glycogen
plasma: B. Gluconeogenesis
C. Breakdown of glycogen
A. Protective value D. Breakdown of proteins
B. Coagulation threshold E. Breakdown of lipids
C. Critical micelle concentration
D. Hydrophilic-lipophilic balance 161. A patient has developed
E. Volume of sol coagulated by 1 mol of megaloblastic anemia on a background
the electrolyte substance of alcoholic cirrhosis. The main cause of
anemia in this patient is the following vi-
156. After an insulin injection a 45-year- tamin deficiency:
old woman with a long history of di-
abetes mellitus has developed weakness, A. Folic acid
paleness, palpitation, anxiety, double visi- B. Lipoic acid
on, numbness of lips and the tip of tongue. C. Biotin
Blood glucose is at the rate of 2,5 mmol/l. D. Thiamin
What complication has developed in the E. Pantothenic acid
patient?
162. Inflorescence of greater plantain
A. Hypoglycemic coma grows out at apex, the main axis is long,
B. Hyperosmolar coma and flowers are sessile. This type of
C. Hyperglycemic coma inflorescence is called:
D. Hyperketonemic coma
E. Uremic coma A. Spike
B. Panicle
157. Morphological analysis of poplar C. Spadix
inflorescence showed that it is a simple D. Capitulum
monopodial inflorescence: main axis is E. Thyrsus
drooping, the flowers are sessile, uni-
sexual. Specify the type of inflorescence: 163. In order to identify the cations
of zinc (II) an analytical chemist used
A. Catkin the reagent solution of hexacyanoferrate
B. Head (II) potassium (Pharmacopeia reaction).
C. Capitulum What colour precipitate is formed in this
D. Cyme reaction?
E. Panicle
A. White
158. Specify the compound with the most B. Yellow
pronounced basic properties: C. Black
D. Green
E. Red
120
krok123.in.ua
Krok 1 Pharmacy 2012 18
164. A patient was found to have an in the urine of a young man. In what
increased blood serum LDH-1 activity. In case a healthy person may present a sli-
which organ is the pathological process ght proteinuria?
localized?
A. After exercise
A. Heart B. In the resting state
B. Liver C. During sleep
C. Kidneys D. In the state of psychoemotional exci-
D. Stomach tation
E. Muscles E. After overeating
165. Diaphoretic herbal tea includes di- 170. A male patient was found to have
chasial cymes with light-yellow, oblong, hypovitaminosis P P . What amino acid
wing-like, squamelliferous perianth. The taken with food may partially compensate
flowers are fragrant, yellowish. These the vitamin P P deficiency?
inflorescences belong to:
A. Tryptophan
A. Tilia cordata B. Phenylalanine
B. Viburnum opulus C. Valine
C. Robinia pseudoacacia D. Arginine
D. Mentha piperita E. Methionine
E. Padus avium
171. Emulsions are classified according
166. A patient with alcoholic cirrhosis to the volume concentration of dispersed
complains of general weakness, dyspnea. phase. An emulsion with the concentrati-
He has been found to have decreased on at the rate of 0,1-74,0% vol. relates to
blood pressure, ascites, enlargement of the following group of emulsions:
superficial veins of the anterior abdominal
wall, esophageal varices, splenomegaly. A. Concentrated
What hemodynamic disorder is observed B. Diluted
in the patient? C. Highly concentrated
D. Direct
A. Portal hypertension E. Reversible
B. Left ventricular failure
C. Right ventricular failure 172. Choose a reaction, in which a basic
D. Heart failure salt is formed:
E. Collapse
A. F e(OH)3 + 2HCl
167. A patient with tuberculosis has been B. F e(OH)3 + 3KCl
prescribed some anti-TB preparations. C. 2NaOH + H2 SO4
Which of the following chemotherapeutic D. KOH + H2 SO4
drugs has an effect on the tuberculosis E. NaOH + HCl
pathogen?
173. Ammonia solution has been added to
A. Ftivazide the solution under examination. A black
B. Furacilinum precipitate fell out. This indicates the
C. Methisazonum presence of the following cations in the
D. Sulfadimezinum solution:
E. Phthalylsulfathiazole
A. Mercury (I)
168. What segment of a nephron contains B. Copper (II)
liquid with a maximum concentration of C. Iron (III)
glucose under normal conditions? D. Iron (II)
E. Silver (I)
A. Poximal tubules
B. Medullary thick ascending limb of loop 174. The causative agents of intesti-
of Henle nal infections can grow at refrigerator
C. Inner medullary portion of thin temperatures, which may cause infection
descending limb of loop of Henle in people. What type of temperature opti-
D. Distal convoluted tubule mum do these microorganisms relate to?
E. Inner medullary collecting duct
169. Laboratory analysis revealed protein
121
krok123.in.ua
Krok 1 Pharmacy 2012 19
124
krok123.in.ua
Krok 1 Pharmacy 2013 1
A. Loradatine A.
B. Diphenhydramine
C. Promethazine
D. Suprastinum
E. Tavegil (Clemastine)
B.
10. Solutions of some electrolytes are
used as medications. What is the maxi-
mum value of the isotonic coefficient for
MgSO4 solution?
C.
A. 2
B. 4
C. 3
D. 5
E. 7 D.
126
krok123.in.ua
Krok 1 Pharmacy 2013 3
127
krok123.in.ua
Krok 1 Pharmacy 2013 4
local temperature, increased tissue turgor. has been found to include flagellated
What local blood circulation disorder are bacteria. What is the most likely mi-
these presentations typical for? croorganism that has been found by the
doctor?
A. Arterial hyperemia
B. Venous hyperemia A. Trihomonas vaginalis
C. Thrombosis B. Leishmania donovani
D. Embolism C. Trypanosoma gambiense
E. Ischemia D. Trihomonas hominis
E. Lamblia intestinalis
40. Microscopic examination of a
perennial stem revealed the secondary 46. A patient has been taking diclofenac
integumentary tissue that was formed as a sodium for a long time. The family physi-
result of cell division of: cian withdrew this drug and administered
celecoxib. What disease was the reason
A. Phellogen for the drug substitution?
B. Procambium
C. Cambium A. Peptic ulcer
D. Pericycle B. Bronchial asthma
E. Protoderma C. Urolithiasis
D. Arterial hypertension
41. What synthetic drug of the hydrazi- E. Chronic hepatitis
de group is typically prescribed for
pulmonary tuberculosis? 47. The rate of a chemical reaction DOES
NOT DEPEND on the concentration of
A. Isoniazid the reactants. Specify the order of such
B. Rifampicin reaction:
C. Acyclovir
D. Metronidazole A. Zeroth
E. Doxycycline hydrochloride B. First
C. Second
42. A female student with a cold has D. Third
been prescribed an antipyretic medicati- E. Fraction
on. Specify this drug:
48. A patient has been diagnosed with
A. Paracetamol bronchial asthma. Specify a medicament
B. Ascorbic acid that can be administered for asphyxiation:
C. Oxytocin
D. Famotidine A. Salbutamol
E. Cyanocobalamin B. Diclofenac sodium
C. Paracetamol
43. Specify the drug that constricts pupils D. Anapriline
and reduces intraocular pressure: E. Acetylcysteine
A. Pilocarpine hydrochloride 49. Sulfanilamides inhibit the growth and
B. Fenofibrate development of bacteria. The mechanism
C. Nitrazepamum of their action is based on the impairment
D. Atropine sulfate of the following acid synthesis:
E. Dithylinum
A. Folic
44. A plant under study has stipules fused B. Lipoic
together and thus forming a tight tube - C. Nicotinic
ochrea, that is a diagnostic feature of the D. Pantothenic
following family: E. Pangamic
A. Polygonaceae 50. Sulfanilamides are widely used as
B. Gramineae bacteriostatic agents. The mechanism of
C. Rosaceae antimicrobial action of sulfanilamides is
D. Papaveraceae based on their structural similarity to:
E. Clusiaceae
45. A 42-year-old female has foamy-
purulent vaginal discharges. The smear
stained by Romanovsky-Giemsa’s method
129
krok123.in.ua
Krok 1 Pharmacy 2013 6
C.
A. Sodium bicarbonate solution
B. Iron (III) chloride solution
C. Sodium hydroxide solution
D. Sodium chloride solution
E. Bromine solution
D. 69. Ethane is the product of the following
reaction:
E.
A. Cu(OH)2 A. Concentrated
B. HBr B. Diluted
C. F eCl3 C. Highly concentrated
D. KMnO4 D. W/O type
E. Ag2 O E. O/W type
79. A patient was found to have a 84. Bacteriological inspection of disi-
tumor of the pancreatic head, which is nfection quality at a pharmacy revealed
accompanied by the impaired patency a microorganism in an utility room (in the
of the common bile duct. Blood test wi- sink). The microorganism has the followi-
ll reveal an increase in the following ng properties: mobile nonspore-forming
substance level: gram-negative bacteria that form capsular
substance, grow well on ordinary nutrient
A. Bilirubin media, secrete the blue-green pigment.
B. Urea This microorganism is most likely to be
C. Hemoglobin of the following genus:
D. Insulin
E. Adrenaline A. Pseudomonas
B. Proteus
80. On the 2nd day after developing acute C. Clostridium
inflammation of the knee joint, the patient D. Shigella
exhibits the joint enlargement, swelling of E. Vibrio
the skin. At what stage of inflammation
are these signs typically observed? 85. Microbiological analysis of medicinal
raw materials revealed capsular bacteria.
A. Exudation What stain method was used to detect the
B. Alteration capsules?
C. Proliferation
D. Regeneration A. Gin’s
E. Sclerosis B. Ziehl-Neelsen’s
C. Neisser’s
81. Quite often, the soil may contain a D. Gram’s
number of pathogenic microorganisms. E. Ozheshko’s
The causative agents of the following di-
sease may stay viable in the soil for a long 86. At pH value 5,0 and isoelectric
time: point 4,0, the protein will migrate
toward the following electrode during
A. Anthrax electrophoresis:
B. Diphtheria
C. Viral hepatitis A. Anode
D. Pertussis B. Cathode
E. Dysentery C. Calomel
D. Silver chloride
82. As a result of an accident (snakebi- E. Platinum
te) a male patient has the following blood
values: Hb- 80 g/l, RBC- 3, 0 · 1012/l; WBC- 87. The intracellular metabolism of
5, 5·109/l. What type of anemia is observed glycerol starts with its activation. What
in this case? compound is formed as a result of the first
reaction of its conversion?
A. Hemolytic
B. Folic acid-deficiency A. Alpha-glycerolophosphate
C. Posthemorrhagic B. Pyruvate
D. Aplastic C. Lactate
E. Iron-deficiency D. Choline
E. Acetyl coenzyme A
83. Emulsions of 0,1 - 74% dispersed-
phase volume relate to: 88. A 40-year-old patient has a history of
bronchial asthma and bradyarrhythmia.
In order to eliminate bronchospasm, the
drugs of the following pharmacological
group should be administered:
133
krok123.in.ua
Krok 1 Pharmacy 2013 10
C.
A. 8
B. 2
C. 4
D. 6
E. 16 D.
102. This scheme of nitroalkane synthesis
is called the reaction of:
E.
A. Konovalov
B. Zinin
C. Kucherov 105. Halogen atoms in an organic
D. Tishchenko compound can be detected by means of:
E. Chichibabin
A. Beilstein test
103. A hospital admitted a patient wi- B. Molisch’s test
th arterial hypertension induced by renal C. Bayer’s test
artery stenosis. The patient complains of D. Lucas’ test
persistent nausea and headache. The main E. Iodoform test
element in the pathogenesis of hypertensi-
on is the activation of the following 106. Before a surgical operation, a
system: surgeon treated his hands with an alcohol-
containing solution. Which group of drugs
does this solution relate to?
135
krok123.in.ua
Krok 1 Pharmacy 2013 12
C.
D. NH3
E.
A.
C.
A. Acetaldehyde hydrazone
B. Acetaldehyde oxime
C. Acetaldimine
D. Acetaldehyde phenylhydrazone
D. E. Acetaldehyde semicarbazone
112. Select a conjugated diene from the li-
st of diene hydrocarbons:
A. CH2 = CH CH = CH2
B. CH2 = C = CH CH3
C. CH2 = CH CH2 CH = CH2
D.
E.
136
krok123.in.ua
Krok 1 Pharmacy 2013 13
A. Lactase
A. Remission B. Maltase
B. Relapse C. Aldolase
C. Latent period D. Hexokinase
D. Recovery E. Glycosidase
E. Prodromal stage
140. A patient has developed megaloblastic
135. An older patient exhibits low levels anemia on a background of alcoholic ci-
of red blood cells and hemoglobin in rrhosis. The main cause of anemia in this
blood, but the color index is 1,3. Blood patient is the following vitamin deficiency:
smear analysis revealed megaloblasts.
What type of anemia is observed in this A. Folic acid
case? B. Lipoic acid
C. Biotin
A. B12 -folic acid deficiency D. Thiamin
B. Iron-deficiency E. Pantothenic acid
C. Acquired hemolytic
D. Hereditary hemolytic 141. The fruit of black locust is dry, formed
E. Chronic posthemorrhagic of a single carpel, dehisces by the ventral
and dorsal sutures on two sides, the seeds
136. Morphological analysis of poplar are attached along the ventral suture. Such
inflorescence showed that it is a simple fruit is called:
monopodial inflorescence: main axis is
drooping, the flowers are sessile, uni- A. Legume
sexual. Specify the type of inflorescence: B. Siliqua
C. Follicle
A. Catkin D. Capsule
B. Head E. Silicula
C. Capitulum
D. Cyme 142. At a chemical analytical laboratory, a
E. Panicle technician examines a solution of the VI
analytical group cations. After the additi-
137. During the morphologic analysis of on of ammonium thiocyanate and amyl
various plant leaves the students found alcohol, the organic layer turned blue.
the leaves, whose length of the leaf blade What cation is present in the solution?
is 5 times more than its width. Specify the
shape of the leaf blade: A. Co2+
B. Ni2+
A. Linear C. Cu2+
B. Elliptical D. Hg 2+
C. Lanceolate E. Cd2+
D. Ovoid
E. Reniform 143. Microscopic study of soybean seeds
stained with Sudan III revealed some
138. Food rich in carbohydrates at fi- droplets of various sizes. They are:
rst increases the blood sugar and then
decreases its rate due to the insulin acti- A. Lipids
on. What process is activated by this B. Proteins
hormone? C. Starch
D. Inulin
A. Synthesis of glycogen E. Glycogen
B. Gluconeogenesis
C. Breakdown of glycogen 144. An analytical chemist determi-
D. Breakdown of proteins nes sodium cations by ion-exchange
E. Breakdown of lipids chromatography. In order to prepare the
cation-exchange resin in the H + form, the
139. After drinking milk a 1-year-old chi- analyst uses:
ld developed diarrhea, flatulence. The
baby is likely to have the deficiency of A. HCl
the following enzyme: B. CH3 COOH
C. C2 H5 OH
D. H3 P O4
E. CH3 OH
139
krok123.in.ua
Krok 1 Pharmacy 2013 16
156. Colloid silver preparations 161. To isolate the lead (II) chloride from
Protargolum and Collargolum are wi- the other cations of the II analytical group
dely used in medical practice as bacteri- in the systematic analysis, the chloride
cidal drugs. In addition to the active precipitate should be processed with:
ingredients, these drugs contain protein
compounds. What is the function of A. Hot water
proteins in these preparations? B. Ammonia solution
C. Nitric acid solution
A. Prevention of coagulation of the colloi- D. Acetate acid solution
dal solution E. Alkali solution
B. Prolongation of shelf-life
C. Reduction of the side effects 162. What reagent is used to separate
D. Improvement of the drug technology the cations of copper (II) and mercury
E. Potentiation of the bactericidal action from the other cations of the VI analytical
of silver group?
157. Therapeutic preparations for topi- A. Sodium thiosulfate
cal use (transdermal, vaginal, etc.) do not B. Sodium sulfate
require sterility. However, the total permi- C. Bromine water
ssible number of microbial cells and fungi D. Potassium sulfide
in 1 g (ml) of a drug should not exceed: E. Excess of the concentrated ammonia
solution
A. 100
B. 10 163. For cultivation of Brucella, pure
C. 500 cultures should be incubated in CO2 enri-
D. 1000 ched atmosphere. What type of breathing
E. 10 0000 is typical for Brucella?
141
krok123.in.ua
Krok 1 Pharmacy 2013 18
A. Radical is called:
B. Central phloem
C. Central xylem A. Capitulum
D. Bilateral B. Cyme
E. Collateral C. Raceme
D. Head
167. Antidepressants can increase the E. Calathidium
concentartion of catecholamines in the
synaptic cleft. What is the mechanism of 173. The technology of drug production
action of these drugs? widely uses the phenomena of absorption
and ion exchange. Which of the ions will
A. Inhibition of monoamine oxidase be selectively adsorbed on the surface of
B. Activation of monoamine oxidase a silver chloride crystal from an aqueous
C. Inhibition of xanthine oxidase solution?
D. Activation of acetylcholinesterase
E. Inhibition of acetylcholinesterase A. Ag +
B. Cu2+
168. The figwort family Scrophulariaceae C. NO3−
includes a biennial plant up to 1,5 m hi- D. H +
gh, with golden-yellow flowers gathered E. OH −
in spiked inflorescences. The flowers have
five stamens. Specify this plant: 174. A patient with acute myocarditis
exhibits rapid fatigability, shortness of
A. Verbascum flomoides breath, edemata of legs, hepatomegaly.
B. Digitalis purpurea Classify the type of heart failure by the
C. Digitalis grandiflora mechanism of its development:
D. Digitalis lanata
E. Digitalis Ferruginea A. Myocardial
B. Overload
169. Permanganometric titration of C. Compensated
hydrogen peroxide is carried out in the D. Subcompensated
following medium: E. Combined
A. Sulfate 175. A child with PKU has an unpleasant
B. Alkaline mouse-like odor, growth retardation,
C. Nitrate mental retardation.These symptoms are
D. Hydrochloric associated with the high concentration of
E. Alcohol the following substance in blood:
170. Quantitative analysis of zinc salts A. Phenylpyruvic acid
is performed by method of trilonometry. B. Glucose
What indicator is used for this purpose? C. Cholesterol
D. Adrenaline
A. Eriohrome black T E. Uric acid
B. Phenolphtalein
C. Methyl black 176. After a contact with a person havi-
D. Potassium dichromate ng an infectious diseases, the disease
E. Thymol blue pathogens entered the patient’s body and
started to multiply, but the symptoms of
171. In response to the administration the disease were not yet observable. What
of protein drugs, a patient developed an period of the disease is this typical for?
allergic reaction. The development of the
allergic reaction is caused by the increased A. Latent
synthesis of the following compound: B. Prodromal
C. Manifest illness stage
A. Histamine D. Clinical outcome
B. Choline E. Relapse
C. Adrenaline
D. Histidine 177. Extraction is commonly used
E. Serotonin in pharmacy for separating mixtures,
increasing the concentration of any solute
172. Astragalus dasyanthus has sessile and extracting lipophilic compounds from
flowers gathered into inflorescences with a the herbal material. This process is based
short thick axis. This kind of inflorescence
142
krok123.in.ua
Krok 1 Pharmacy 2013 19
on: A. Acetyl-CoA
B. Isocitrate
A. Nernst distribution law C. Lactate
B. Konovalov’s first law D. Malate
C. Dalton’s second law E. Ketoglutarate
D. Third law of thermodynamics
E. Hess’s Law 183. Allantoic fluid of a chicken embryo
contaminated with nasopharyngeal flush
178. The rate of extraction of a drug of a patient was found to contain a virus.
substance depends on the value of its di- What diagnostic agents should be used to
stribution coefficient. If the distributed identify it?
substance is characterized by different
rates of dissociation or association in di- A. Standard antiviral sera
fferent phases, the distribution coefficient B. Viral diagnosticums
is calculated by: C. Serum preparations
D. Diagnosticums produced of standard
A. Shilov-Lepin equation virus strains
B. Nernst distribution law E. Polyvalent immune diagnostic sera
C. Gibbs’ phase rule
D. The first Raoult’s law 184. A patient complains of severe
E. Van’t Hoff rule abdominal pain, cramps, blurred vision.
His relatives exhibit the same symptoms.
179. A male patient developed fever up The urine is of red colour. The patient has
to 40o C, there are vomiting, diarrhea, been hospitalized for acute intermittent
the patient is in grave condition. Blood porphyria. This disease might have been
osmolality is 270 mOsm/l. What disorder caused by the impaired synthesis of the
of water-salt metabolism is observed in following compound:
the patient?
A. Heme
A. Hypoosmolar hypohydration B. Insulin
B. Isoosmolar hypohydration C. Bile acids
C. Hyperosmolar hypohydration D. Prostaglandins
D. Isoosmolar hyperhydration E. Collagen
E. Hypoosmolar hyperhydration
185. Common nettle, hop, black
180. A 40-year-old male presented to elderberry relate to the plants that require
the endocrinology department with di- soils rich in nitrogen compounds, that is,
sproprortionate enlargement of limbs, such plants are called:
mandible and nose. These manifestations
are caused by the overproduction of the A. Nitrophytes
following hormone: B. Nitrophobes
C. Calciphiles
A. Somatostatin D. Calciphobes
B. Corticotropin E. Halophytes
C. Aldosterone
D. Adrenaline 186. A 40-year-old female farmworker
E. Vasopressin has been diagnosed with brucellosis and
administered causal chemotherapy. What
181. Surfactants are commonly used in group of drugs will be used for this
pharmaceutical production. What kind of purpose?
surfactant is potassium oleate?
A. Antibiotic
A. Anionic B. Donor immunoglobulin
B. Cationic C. Inactivated therapeutic vaccine
C. Nonionic D. Polyvalent bacteriophage
D. Amphoteric E. Antitoxic serum
E. None of the above
187. A male received a radiation dose of
182. Diabetes and starvation cause the 30 Gy. He presents with necrotic angi-
excess production of ketone bodies that na, disorders of the gastrointestinal tract.
are used as an energy source. They are Blood tests revealed anemia, leukopenia
produced from the following compound: and thrombocytopenia. What phase of
acute radiation syndrome is observed in
143
krok123.in.ua
Krok 1 Pharmacy 2013 20
144
krok123.in.ua
Krok 1 Pharmacy 2013 21
145
krok123.in.ua
Кrok 1 Pharmacy (англомовнi студенти) 2014 рiк 1
A. Restrictive A. Tetracytic
B. Obstructive B. Diacytic
C. Central C. Anisocytic
D. Peripheral D. Anomocytic
E. Thoracic diaphragm E. Paracytic
10. What naloxone indications are 15. Аrctostaphylos uva ursi, Vaccini-
there? um vitis ideae, Vaccinium myrtillus life
forms can be defined as:
A. Narcotic analgetics acute poisoning
B. Heavy metals poisoning A. Small shrub (fruticulus)
C. Cardiac glycosides poisoning B. Vine
D. Ergot alkaloids poisoning C. Grass
E. Atropine sulphate poisoning D. Shrub (frutex)
E. Subshrub (suffrutex, semifrutex)
11. During gastric secretory functi-
on research decrease of hydrochloric 16. In the course of plant cells
acid concentration in gastric juice was treatment with phloroglucinol wi-
detected. What enzyme will be less th concentrated sulfuric acid their
active in such a condition? cell walls became crimson-red, which
means:
A. Pepsin
B. Amylase A. Lignification
C. Lipase B. Suberization
D. Dipeptidase C. Mucification
E. Hexokinase D. Cutinization
E. Mineralization
12. What reagent does p-aminobenzoic
acid amino group react with? 17. When root was being studied under
microscope, root hairs were detected,
which are cell growths of:
A. Epiblema
B. Epidermis
A. HCl C. Endoderm
B. NH4 OH D. Exoderm
C. NaOH E. Mesoderm
D. CH3 COONa
18. Pharmacy sells glaucine hydrochlori-
E. KCN de to the patient with chronic bronchi-
13. The patient with hepatic colic tis. What common side effect should
has been prescribed spasmolytic of he be warned about?
muscarinic receptor antagonists group A. Decrease of arterial pressure
as a part of his complex therapy. What B. Excitation of central nervous system
drug is it? C. Disruption of cardiac rate
A. Atropine D. Increase of intraocular pressure
B. Proserin E. Allergic skin rashes
C. Galantamine 19. Potassium iodide solution has been
D. Dithylin added to the solution containing cati-
E. Benzohexonium
ons of the sixth analytical group (acid-
14. Microscopy of monocotyledon base classification). It resulted in red
leaf epidermis revealed that stomatal precipitate soluble in excess of reagent.
complex has four accessory cells. That What cations are present in the soluti-
means stomatal apparatus belongs to on?
the following type:
147
krok123.in.ua
Кrok 1 Pharmacy (англомовнi студенти) 2014 рiк 3
B.
+ E.
CH3 − CH2
C.
−
CH3 − CH2
36. At the sixth month of pregnancy
D. the female patient has been diagnosed
− with severe iron-deficiency anemia. Di-
OH agnostic character was the appearance
of the following in blood:
E.
N̈H3
149
krok123.in.ua
Кrok 1 Pharmacy (англомовнi студенти) 2014 рiк 5
153
krok123.in.ua
Кrok 1 Pharmacy (англомовнi студенти) 2014 рiк 9
D.
A.
E.
B.
D.
E.
154
krok123.in.ua
Кrok 1 Pharmacy (англомовнi студенти) 2014 рiк 10
A. A.
2C2 H5 Cl+2Na → CH3 CH2 −CH2 CH3 +NaCl
B.
hv
B. C2 H6 + Cl2 −→ C2 H2 Cl + HCl
C.
C2 H5 OH + Cl → CH3 CH2 Cl + H2 O
C. D.
(HOH)
C2 H5 Cl + NaOH −−−−→
D. E.
(C2 H5 OH)
C2 H5 Cl + NaOH −−−−−−−→
H2 C = CH2 + H2 O + NaCl
E.
A. CO2 + CO + H2 O
B. 2CO2 + H2
C. H2 O + CO2
D. 2CO + H2 + O2
E. C2 H2 + 2O2
76. Point out the substance produced
during the following reaction:
HOH,Hg 2+
CH ≡ CH −−−−−−−→?
A. Ethanal
B. Ethanol
C. Propionaldehyde
D. Propanone
E. Acetate acid
77. When substances interact according
to the scheme given in
155
krok123.in.ua
Кrok 1 Pharmacy (англомовнi студенти) 2014 рiк 11
B.
D.
C.
E.
D.
A. Acetaldehyde hydrazone
B. Acetaldoxime
C. Acetaldimine A. NaNO2 (HCl)
D. Acetaldehyde phenylhydrazine B. K2 Cr2 O7
E. Acetaldehyde semicarbazone C. NaNO3 (H2 SO4 )
D. KOH
80. Point out the product of the reacti- E. Cu(OH)2
on given below:
83. Which of the alcohols given in
156
krok123.in.ua
Кrok 1 Pharmacy (англомовнi студенти) 2014 рiк 12
157
krok123.in.ua
Кrok 1 Pharmacy (англомовнi студенти) 2014 рiк 13
prolonged period of dormancy. How 115. Milk intake has resulted in the
can this kind of disease progression be one-year-old child having diarrhea and
qualified? abdominal distension. What enzyme
deficiency does the child have?
A. Relapse
B. Remission A. Lactase
C. Recovery B. Maltase
D. Latent period C. Aldolase
E. Prodromal stage D. Hexokinase
E. Glycosidase
111. The 55-year-old female pati-
ent has developed a case of acute 116. The 56-year-old patient has
pancreatitis caused by greasy food. developed megaloblastic anemia in the
What is the main pathogenesis step of course of alcoholic cirrhosis. What vi-
this disorder? tamin deficiency is the main cause of
anemia in this patient?
A. Premature activation af enzymes in
gland ducts and cells A. Folic acid
B. Pancreatic juice deficiency B. Lipoic acid
C. Low bile production in liver C. Biotin
D. Fats digestion disruption D. Thiamine
E. Acute bowel obstruction E. Pantothenic acid
112. As the result of taking herbal 117. During morphological analysis of
medicine the 30-year-old patient has lily-of-the-valley (Convallaria majalis)
developed anaphylactic allergic reacti- leaf it was noted that lamina has wide
on and blood leukocytosis. What kind elliptic shape and numerous veins are
of leukocytosis is characteristic of this parallel to leaf margin and merge only
case? at the leaf point. What is this venation
type called?
A. Eosinophilia
B. Monocytosis A. Arcuate
C. Lymphocytosis B. Parallel
D. Basophilia C. Palmate
E. Heutrophilia D. Pinnate-reticulate
E. Dichotomous
113. During morphological descripti-
on of common periwinkle it was defi- 118. In the process of silver cations
ned that it has shoot that trails on the identification reaction HCl and then
ground and takes root. It allows to ammonia solution have been added to
chatacterize such shoot as: the solution. What compound has been
produced as a result?
A. Creeping
B. Recumbent A. [Ag(NH3 )2 ]Cl
C. Twining B. [Ag2 (NH3 )3 ]Cl
D. Scandent C. AgOH
E. Tenent D. AgCl
E. [Ag(NH3 )3 ]Cl
114. L-DOPA and its derivatives
are used in treatment of Parki- 119. In chemico-analytical laboratory
nson’s disease. What aminoacid is this a specialist studies the mixture of the
substance made of? 5th analytical group cations. When thi-
ocyanate ions are added the solution
A. Tyrosine becomes red-colored. This analytical
B. Asparagine effect indicates presence of the followi-
C. Glutamate ng cation:
D. Tryptophan
E. Arginine
160
krok123.in.ua
Кrok 1 Pharmacy (англомовнi студенти) 2014 рiк 16
A. F e3+ A. Tetradynamous
B. F e2+ B. Didynamous
C. Mg 2+ C. Diadelphous
D. Bi3+ D. Monadelphous
E. Mn2+ E. Polydelphous
120. Dispensing chemist conducts 124. The dispensing chemist has been
quantitative determination of studying properties of certain disperse
pharmaceutical substance with system classes, namely, aerosols. What
restorative properties through direct optical phenomenon is characteristic
bromate titration. What solution is the of this disperse system class?
titrant?
A. Light scattering
A. Potassium bromate B. Light absorption
B. Iodine solution in potassium iodide C. Opalescence
C. Sodium thiosulfate D. Light reflection
D. Potassium iodide E. Light refraction
E. Chloride acid
125. Selective solvents are used in
121. In the chemico-analytical laboratories and factories to isolate
laboratory the dispensing chemist and refine essential oils, alkaloids,
studies solution of anion mixture. antibiotics and other pharmaceutical
When antipyrin solution is added it subctances. This process is called:
becomes emerald-green colored. This
analytical effect signifies presence of A. Extraction
the following anions: B. Sedimentation
C. Coagulation
A. Nitrite D. Flocculation
B. Nitrate E. Flotation
C. Acetate
D. Tartrate 126. When rare dosage forms are
E. Citrate produced, colloid surfactants are
added to increase certain components
122. In microbiology class students had solubility. What physicochemical
been growing pure bacterial culture. phenomenon is this process based on?
Bacterial inoculation of solid medium
was performed to obtain separate visi- A. Solubilization
ble colonies, resulting in two coloni- B. Coagulation
es, R-type and S-type, grown in C. Extraction
thermostat after one day of incubation. D. Diffusion
What microorganism properties were E. Sedimentation
described by students? 127. A pharmacist has been adding
A. Cultural small portions of electrolyte to silver
B. Tinctorial chloride sol, with resulting coagulati-
C. Biochemical on occuring under higher electrolyte
D. Morphologic concentration, if compared to single
E. Antigenic instance of adding electrolyte. This
phenomenon is called:
123. Androecium of Brassica oleracea
flower has six stamens, with four A. Sol acclimatization
stamens of inner circle longer than two B. Antagonism
stamens of outer circle. What is this C. Synergism
type of androecium called? D. Additivity
E. Desensitization
128. Ketoacidosis occurs during
starvation. What metabolite blood
concentration increase is symptomatic
161
krok123.in.ua
Кrok 1 Pharmacy (англомовнi студенти) 2014 рiк 17
162
krok123.in.ua
Кrok 1 Pharmacy (англомовнi студенти) 2014 рiк 18
A. Viruses
B. Fungi A. Type I pancreatic diabetes
C. Bacteria B. Type II pancreatic diabetes
D. Protozoa C. Diabetes insipidus
E. Rickettsia D. Steroidogenic diabetes
E. Glycogenosis
137. Emulsions are widely used in
pharmaceutical practice. What is the 142. The patient with mushroom poi-
process of spontaneous merging of soning has developed the following
drops in emulsions called? symptoms: yellow coloring of skin and
sclera, dark-colored urine. Hemolytic
A. Coalescence jaundice was diagnosed. What pigment
B. Flocculation causes such coloring of the patient’s
C. Sedimentation urine?
D. Flotation
E. Coagulation A. Stercobilin
B. Conjugated bilirubin
138. Streptomycin and other ami- C. Biliverdin
noglycosides by binding with 30S- D. Unconjugated bilirubin
subunit of ribosome prevents E. Verdohemoglobin
formylmethionyl-tRNA joining. What
process is disrupted due to this effect? 143. Phenomenon of decreasing system
volume resulting from polymer swelli-
A. Translation initiation ng is called:
B. Translation termination
C. Transcription initiation A. Contraction
D. Transcription termination B. Solvation
E. Replication initiation C. Sedimentation
D. Dissolution
139. Production of injections in E. Coagulation
pharmacies requires srtict control of
sterilization quality. What is placed 144. If the amount of high-molecular
in autoclave sterilization chamber to substance added to the given sol is
ensure proper control? extremely small, it is possible its stabi-
lity will decrease, instead of increase.
A. Ampoule with microbe spores What is this phenomenon called?
B. Ampoule with staphylococcus
culture A. Sensitization
C. Ampoule with colibacillus culture B. Solubilization
D. Ampoule with fungi spores C. Syneresis
E. Ampoule with viruses D. Sedimentation
E. Synergism
140. The patient has hypovitaminosis
PP. What amino acid taken with meals 145. Ammonia solution has been
partially compensates patient’s need added to the solution being studi-
for vitamin PP? ed. Black precipitate has formed.
That means the following cations are
A. Tryptophan present in the solution:
B. Phenylalanine
C. Valine A. Mercury (I)
D. Arginine B. Copper (II)
E. Methionine C. Iron (III)
D. Iron (II)
141. The 13-year-old female patient E. Silver (I)
having suffered from measles complai-
ns of dry mouth, thirst, body weight 146. Silver nitrate solution has been
loss, polyuria, her glucose concentrati- added to the solution containing ani-
on in blood is 16 mmol/l. What disease ons of the first analytical group. It
can be suspected?
163
krok123.in.ua
Кrok 1 Pharmacy (англомовнi студенти) 2014 рiк 19
168
krok123.in.ua
Кrok 1 Pharmacy (англомовнi студенти) 2014 рiк 24
169
krok123.in.ua
Krok 1 Pharmacy 2015 1
1. Calculation of thermal effects of below are all of the same molarity, the hi-
chemical reactions at pharmaceutical ghest boiling temperature will belong to:
production is based on the Hess law stati-
ng, that reaction thermal effect is determi- A. Sodium sulfate
ned by: B. Promedol (Trimeperidine)
C. Nicotinamide
A. Initial and final state of system D. Resorcin
B. Mechanism by which the chemical E. Iodine
change occurs
C. Route by which the chemical change 7. Which of these formulas corresponds wi-
occurs th acetoacetic acid?
D. Number of intermediate stages
E. Process duration
A.
2. A victim of a traffic accident is hospi-
talized at a resuscitation unit. Objectively:
the patient is unconscious, BP is 90/60 mm
Hg, high blood content of creatinine and B.
urea is observed, diurnal diuresis is 80 ml.
Characterize the patient’s diurnal diuresis:
C.
A. Anuria
B. Oliguria
C. Polyuria
D. Pollakiuria D.
E. Nocturia
3. In snake venom there is a substance
that causes erythrocyte hemolysis when E.
it is introduced into a human organi-
sm. Blood test revealed a large amount 8. Universal system of biological oxidati-
of lysolecithin (lysophosphatidylcholi- on of nonpolar compounds (numerous
ne). What enzyme leads to accumulating drugs, toxic agents, steroid hormones,
lysolecithin in blood? cholesterol) is microsomal oxidation.
A. Phospholipase A2 Name the cytochrome that is included in
B. Phospholipase A1 oxygenase chain of microsomes:
C. Phospholipase C A. Cytochrome Р-450
D. Phospholipase D B. Cytochrome C
E. Neuraminidase C. Cytochrome A3
4. Microbe survial in environment is faci- D. Cytochrome A
litated by spore formation. What mi- E. Cytochrome C1
croorganisms of those listed below are 9. A patient with chronic constipation had
spore formers: been prescribed bisacodyl. After 3 weeks
A. Clostridium of treatment, the patient noticed a reducti-
B. Bacteroides on of laxative effect. This is caused by the
C. Staphylococcus development of the following side-effect:
D. Peptococcus A. Tolerance
E. Peptostreptococcus B. Dependence
5. Bacteria rapidly become resistant C. Sensibilization
to drugs in the course of antibacterial D. Cumulation
treatment. What structural components of E. Dysbacteriosis
bacteria provide for their resistance? 10. A newborn child born from Rh-
A. R-plasmids negative mother in the result of her thi-
B. Spores rd pregnancy presents with gradually
C. Capsule worsening jaundice, irritated central
D. Flagella nervous system, anemia. What type of
E. Volutine granules jaundice does the infant suffer from?
A. Gas embolism 18. When root was being studied under mi-
B. Fat embolism croscope, root hairs were detected, which
C. Air embolism are cell growths of:
D. Cellular embolism
E. Thromboembolism A. Epiblema
B. Epidermis
13. The patient has been hospitalised with C. Endoderm
pneumonia. What kind of respiratory fai- D. Exoderm
lure does the patient have? E. Mesoderm
171
krok123.in.ua
Krok 1 Pharmacy 2015 3
C.
D.
A.
E.
B.
A. D.
B.
E.
176
krok123.in.ua
Krok 1 Pharmacy 2015 8
A. Radicle
A. B. Apical meristem
C. Pericycle
B. Na2 SO4 D. Lateral meristem
C. E. Intercalary meristem
D. NaNH2
E. NaNO3 74. Silver mirror reaction can be
characterized by:
69. Aniline can be converted into the
water-soluble salt through its treatment wi- A. Production of metal
th the solution of: B. Smell
C. Red colouring of precipitate
A. Hydrochloric acid D. Blue colouring of solution
B. Sodium hydroxide E. Decolouration
C. Sodium sulfate
D. Ethanol 75. The end product of starch hydrolysis is:
E. Dimethylamine A. D-glucose
70. Specify the reagent necessary for the B. D-fructose
following transformation: C. Saccharose
D. Maltose
E. D-galactose
76. Specify the electronic effects of the
carboxylic group (-СООН) in a benzoic
acid molecule:
A. NH2 NH2
B. NH2 OH A. -I, -M
C. CH3 NH2 B. -I
D. C6 H5 NHNH2 C. +I, -M
E. NH3 D. +I
E. -I, +M
71. One of the cyclic glucose forms is as
follows: 77. Which of the alcohols given below
produces acetone when oxidated?
A. α-D-glucopyranose A.
B. β -D-glucopyranose
C. α-L-glucopyranose
D. α-D-glucofuranose B.
E. β -D-glucofuranose
72. Select the formula for pentene-2 from C.
the list:
D.
A. CH3 − CH2 − CH = CH − CH3
B. CH3 − CH2 − CH2 − CH2 − CH3 E.
C. CH3 − CH2 − CH2 − CH = CH2
D. CH3 − CH2 − CH2 − CH3
E. CH3 − CH = CH − CH3 78. What reagent allows to simultaneously
detect both aldehyde group and glycol
73. The study of the main root ontogenesis fragment presence in glucose molecule?
shows that it has developed from:
A. Cu(OH)2
B. Br2
C. AlCl3
D. F eCl3
E. KMnO4
177
krok123.in.ua
Krok 1 Pharmacy 2015 9
178
krok123.in.ua
Krok 1 Pharmacy 2015 10
A. Bifidumbacterin A. Lactase
B. Colibacterin B. Maltase
C. Coli-Proteus bacteriophage C. Aldolase
D. Furazolidone D. Hexokinase
E. Lactoglobulin E. Glycosidase
90. A sample of water used in drug 95. A man presents with signs of albi-
production has been sent to a laboratory nism: blonde hair, extreme photosensi-
for sanitary and virological analysis. tivity, impaired vision. What amino acid
Presence of what virus group will be indi- metabolism is disrupted in the patient?
cative of faecal contamination of water and
thus the need for its additional purificati- A. Tyrosine
on? B. Methionine
C. Proline
A. Picornaviridae D. Histidine
B. Herpesviridae E. Valine
C. Orthomyxoviridae
D. Retroviridae 96. In the process of silver cations identi-
E. Flaviviridae fication reaction HCl and then ammonia
solution have been added to the solution.
91. Specify the indicator of the protective What compound has been produced as a
properties of high-molecular compounds result?
of body, which promote the retention of
calcium, phosphate and carbonate in blood A. [g(N 3 )2 ]l
plasma: B. [g2 (N 3 )3 ]l
C. g
A. Protective value D. g l
B. Coagulation threshold E. [g(N 3 )3 ]l
C. Critical micelle concentration
D. Hydrophilic-lipophilic balance 97. Microscopic study of soybean seeds
E. Volume of sol coagulated by 1 mol of the stained with Sudan III revealed droplets
electrolyte substance of various sizes. They are:
92. A 40-year-old man diagnosed with A. Lipids
gastric ulcer has developed the symptoms B. Proteins
anew after a long period of dormancy. Such C. Starch
disease course can be characterized as a: D. Inulin
E. Glycogen
A. Recurrence
B. Remission 98. In a chemico-analytical laboratory a di-
C. Recovery spensing chemist studies the solution of
D. Latency anion mixture. When antipyrin solution is
E. Prodromal phase added to the solution, it becomes emerald-
green in colour. This analytical effect signi-
93. An elderly patient exhibits low levels of fies presence of the following anions:
red blood cells and hemoglobin in blood,
but the color index is 1,3. Blood smear A. Nitrite
analysis revealed megaloblasts. What type B. Nitrate
of anemia is observed in this case? C. Acetate
D. Tartrate
A. B12 -folic acid deficiency E. Citrate
B. Iron-deficiency
C. Acquired hemolytic 99. Selective solvents are used in
D. Hereditary hemolytic laboratories and factories to isolate and
E. Chronic posthemorrhagic refine essential oils, alkaloids, antibiotics
and other pharmaceutical subctances. This
94. After drinking milk a 1-year-old child process is called:
has developed diarrhea and flatulence. The
baby is likely to have the deficiency of the A. Extraction
following enzyme: B. Sedimentation
C. Coagulation
D. Flocculation
E. Flotation
100. Sol of iron (III) hydroxide is positi-
179
krok123.in.ua
Krok 1 Pharmacy 2015 11
102. What indicator is used for fixing the 107. Mass fraction of F e2+ ions in Mohr’s
endpoint of mercurimetric titration? salt can be determined by gravimetric sedi-
mentation method using:
A. Thiocyanate complexes of iron (III)
B. Fluorescein A. NH4 OH
C. Eosin B. Na2 S
D. Murexide C. K3 P O4
E. Potassium chromate D. BaCl2
E. ZnCl2
103. Substrate-linked phosphorylation
occurs in the cycle of tricarboxylic acids. 108. Silver nitrate solution has been added
What compound takes part in this reacti- to the solution containing anions of the fi-
on? rst analytical group. It resulted in yellow
precipitate. That means the following are
A. Succinyl coenzyme A present in the solution:
B. α-ketoglutarate
C. Acetyl coenzyme A A. Arsenite ions
D. Succinate B. Arsenate ions
E. Malate C. Sulphate ions
D. Iodide ions
104. The patient with alcoholic ci- E. Bromide ions
rrhosis complains of general weakness
and dyspnea. The following is revealed: 109. Inhibition of the synthesis of bile aci-
decrease of arterial pressure, ascites, di- ds from cholesterol in liver of an experi-
lation of the superficial veins of the mental animals has caused maldigestion of
stomach anterior wall, esophageal vari- lipids. What is the role of these acids in the
cose veins dilatation, splenomegaly. What enteral lipidic metabolism?
haemodynamics disorder does the patient
suffer from? A. They emulsify dietary lipids
B. They keep balance of alkaline envi-
A. Portal hypertension ronment in the intestines
B. Left ventricular failure C. They participate in the synthesis of lipids
C. Right ventricular failure D. They are part of LDL
D. Cardiac insufficiency E. They activate the formation of chylomi-
E. Collapse crons
105. Pharmacy has received viricides. 110. Name the above-ground sprout modi-
Choose the viricide used for influenza fications that develop from lateral buds,
treatment from the list given below: are situated in leaf angles or inflorescences,
and take part in vegetative reproduction:
180
krok123.in.ua
Krok 1 Pharmacy 2015 12
narcotic analgesic for labor pain relief. gastric ulcer disease was prescribed a
What drug is indicated in this case? medicine as a part of his multimodality
therapy. The medicine lowers acidity of
A. Promedol (Trimeperidine) gastric juice, inhibits + , + -adenosine
B. Morphine triphosphatase, decreases the volume of
C. Papaveretum (Omnopon) gastric secretion and pepsinogen producti-
D. Codeine on. It is a prodrug. Name this medicine:
E. Fentanyl
A. Omeprazole
166. A woman, who during the 5th-10th B. Famotidine
weeks of her pregnancy had been taki- C. Gastrozepin (Pirenzepine)
ng sodium valproate for treatment of D. Ranitidine
her epilepsy, gave birth to a child with E. Phosphalugel (Aluminium phosphate)
pathology of the vertebral column (split
spine). What side effect of the drug caused 171. A cultivated plant has green berrylike
such malformation? fruit and underground sprout modificati-
ons - tubers. The described plant is:
A. Teratogenic
B. Mutagenic A. Solanum tuberosum
C. Embryotoxic B. Соnvalaria majalis
D. Fetotoxic C. Polygonatum odoratum
E. Sensitizing D. Atropa belladonna
E. Solanum lycopersicum
167. A patient with acute bronchitis was
prescribed an expectorant that caused 172. Elongated narrow prismatic crystals
bronchial spasm after the patient had with sharpened points were detected
taken it. What drug of those listed below during microscopic investigation of
can cause such side effect? Convallaria majalis mesophile. These
crystals are:
A. Acetylcysteine
B. Salbutamol A. Styloids
C. Validol (Menthyl isovalerate) B. Druses
D. Platyphyllin C. Crystalline sand
E. Prenoxdiazine (Libexin) D. Cystoliths
E. Perigonium
168. A patient was visiting a pharmacy,
when he suddenly felt unwell. He 173. Students should identify the following
developed palpitations, rapid heart rate, to determine the sex of a flower:
pain in the chest that after several minutes A. Stamens and pistils
spread to the left scapula and left side of B. Flower cup and corolla
the head. What condition should be consi- C. Pedicle and receptacle
dered first? D. Symmetry
A. Ischemic heart disease E. Colour and type of indumentum
B. Peptic gastric ulcer disease 174. A doctor prescribed a herbal drug
C. Dysphagia with flavonoid complex of Silybum mari-
D. Pneumonia anum to a patient suffering from chronic
E. Somatoform autonomic dysfunction hepatitis. This hepatic protector stimulates
169. Prolonged application of broad protein synthesis, normalizes phospholipid
spectrum antibacterial drugs resulted in metabolism, acts as an antioxidant. Name
the patient being hospitalised with di- this drug:
agnosis of candidiasis. What side effect A. Silymarin
of antibiotic therapy has developed in the B. Essentiale
patient? C. Galstena
A. Disbacteriosis D. Thiotriasoline
B. Endotoxic reaction E. Ursodeoxycholic acid
C. Toxic reaction 175. A pharmaceutical manufacture
D. Allergic reaction produces a drug, that is an animal antibi-
E. Formation of resistant microorganism otic. Point out this drug among those listed
strains below:
170. A 37-year-old patient with peptic
186
krok123.in.ua
Krok 1 Pharmacy 2015 18
A. Lysozyme A. Salbutamol
B. Gramicidin B. Metoprolol
C. Novobiocin C. Atenolol
D. Phaseolin D. Anaprilin (Propranolol)
E. Chloramphenicol E. Nebivolol
176. A patient has been receiving 181. A 25-year-old-patient with the II
Theophylline - inhibitor of cyclic adenosi- degree thermal burns addressed a doctor.
ne monophosphate phosphodiesterase - Objectively: there are large blisters on the
for a week. What hormone can increase upper limbs; the blisters are filled with
its action due to such treatment and cause clear exudate containing mostly water and
hyperglycemia? albumines with isolated leukocytes. Name
the type of the exudate:
A. Glucagon
B. Testosterone A. Serous
C. Aldosterone B. Catarrhal (mucous)
D. Insulin C. Fibrinous
E. Estradiol D. Purulent
E. Hemorrhagic
177. To detect anions in a solution by fracti-
onal method a reaction with iron (III) 182. A seed of a legume contains proteins
chloride was performed in acid medium. and fatty oil. Name this legume:
The solution coloured red-violet. What
anion is the cause of such analytical effect? A. Glycine hispida
B. Vaccinium myrtillus
A. Salicylate C. Sinapis alba
B. Chloride D. Astragalus dasyanthus
C. Nitrate E. Datura stramonium
D. Bromate
E. Phosphate 183. Rhizome of a species belonging to
the Asteraceae family is polycephalous,
178. An impression smears of the dead succulent, has lysigenous cavities,
person’s brain and salivary glands revealed accumulates inulin. Such underground
Negri bodies, when coloured with Mann organ is characterisic of:
methyl blue-eosin stain. These results
confirm the presence of the following di- A. Inula helenium
sease: B. Hyoscyamus niger
C. Digitalis grandiflora
A. Hydrophobia D. Sorbus aucuparia
B. Influenza E. Helianthus annuus
C. Parotitis
D. Parainfluenza 184. What drug is used in treatment regi-
E. Encephalitis men for peptic ulcer disease to eliminate
Helicobacter pylori?
179. Preparations of colloid silver -
Protargol (silver proteinate) and Collargol A. Clarithromycin
(colloid silver) - contain proteine B. Tienam
compounds besides their active substance. C. Biseptol
What is the function of proteins in these D. Chloridine
preparations? E. Sulfalene
effect? A. Paresis
B. Paralysis
A. Psychosedatives C. Myasthenia
B. Cholinergic antagonists D. Hyperkinesia
C. Adrenergic drugs E. Tremor
D. Diuretics
E. Emetics 193. A 32-year-old patient with cerebellar
tumor was delivered to an admission room
187. Some drugs have the form of colloid of a hospital. The patient presents with
solutions. What size of dispersed phase ataxia that can be characterized by:
particles corresponds with colloidal di-
spersion? A. Disrupted coordination of movements
B. Involuntary contraction of skeletal
A. 10−7 − 10−9 m muscles
B. 10−5 − 10−7 m C. Increased muscle tone
C. 10−10 − 10−11 m D. Pathological reflexes
D. 10−5 − 10−3 m E. Irregular force and direction of
E. >10−3 m movements
188. A local general practitioner 194. A doctor prescribed diazepam to
recommends taking interferon for a patient with anxiety disorders. What
influenza prevention. What is the mechani- pharmacological effect is the reason for
sm of action of this drug? such a prescription?
A. Blocks virus protein synthesis A. Anxiolytic
B. Blocks virus stripping B. Anticonvulsant
C. Inhibits virion exit from cells C. Anti-inflammatory
D. Prevents adsorption of virus in cell D. Antianginal
receptors E. Hypotensive
E. Disrupts the process of virus assembly
195. A 40-year-old man presents with rapid
189. What cation can be detected with weight gain after he had suffered a severe
Chugaiev’s agent (Dimethylglyoxime)? craniocerebral trauma. At doctor’s exami-
nation the patient’s weight was 125 kg, with
A. Ni2+ his weight being 175 cm. What mechanism
B. Ca2+ of obesity development is the most likely
C. + in this case?
D. Mn2+
E. Co2+ A. Hypothalamic
B. Alimentary
190. Dry many-seeded monocarp fruit C. Hormonal
opens along its ventral suture. It can be D. Hereditary
identified as: E. -
A. Follicle 196. Specify the parameters that
B. Legume characterize the sensitivity of analytical
C. Nutlet reaction:
D. Drupe
E. Capsule A. All the parameters
B. Minimal volume of borderline diluted
191. Determination of silver salts by solution
ammonium thiocyanate titration is C. Borderline dilution
performed in the presence of the following D. Absolute sensitivity
indicator: E. Borderline concentration
A. (NH4 )F e(SO4)2 197. Specify the name of the carbohydrate
B. F eSO4 given in
C. F eCl2
D. (NH4 )2 SO4
E. NH4 SCN
192. After ishemic stroke a 67-year-old
patient developed reduced mobility of the
left leg. Name this condition: according to the systematic nomenclature:
188
krok123.in.ua
Krok 1 Pharmacy 2015 20
A. 5,6,6-trimethyl-1-hepten
B. 2,2,3-trimethyl-6-hepten A. С1
C. 5-tert-Butyl-1-hexene B. С2
D. 2-tert-Butyl-5-hexene C. С3
E. 2,2,3-trimethyl-6-hexene D. С4
198. M. I. Konovalov’s reaction is as E. С5
follows: 200. To chemically distinguish between
A. 4 + HNO3 (diluted) = CH3 NO2 + glycerine and ethylene glycol
H2 O
B. 4 + H2 SO4 = CH3 SO2 OH + H2 O
C. 22 5 Cl + 2Na = C4 H10 + 2NaCl
D. 6 6 + HNO3 (concentrated) = it is nececcary to apply:
C6 H5 NO2 + H2 O
E. 4 + Cl2 = CH3 Cl + HCl
199. Hydrogen atom attaches to the A. KHSO4
following carbon-1-pentene atom during B. CuOH
electrophilic addition of hydrogen bromi- C. Cu(OH)2
de: D. NaOH
E. NaCl
189
krok123.in.ua
Кrok 1 Pharmacy (англомовнi студенти) 2016 рiк 1
cator:
A. Chingamin
A. Diphenylcarbazide B. Rifampicin
B. Potassium chromate C. Ampicillin
C. Eriochrome black T D. Gentamicin
D. Starch E. Biseptol (Co-Trimoxazole)
E. Tropeolin OO
24. A sample section of an axial
19. A patient has been hospitalised body shows a complex consisting of
with pneumonia. What kind of respi- phellogen and its derivatives - cork
ratory failure does the patient have? and phelloderm. This tissue is called:
A. Restrictive A. Periderm
B. Obstructive B. Colenchyma
C. Central C. Sclerenchyma
D. Peripheral D. Epiblema
E. Thoracic diaphragm E. Epidermis
20. In the state of fright the followi- 25. The Mohr method is used to
ng signs can be observed: acute pallor determine mass concentration of
of face, tremor of extremities. What sodium chloride in an isotonic soluti-
kind of ischemia can be observed in on. Titration is carried out with the
such a condition? following indicator:
A. Angiospastic A. Potassium chromate
B. Compression B. Fluorescein
C. Obstructive (thrombus) C. Ammonium iron (III) sulfate
D. Metabolic D. Diphenylcarbazone
E. Obstructive (vascular wall thickeni- E. Ferroin
ng)
26. An injured person exhibits
21. What are the indications for the the following signs at the site of
use of naloxone? trauma: skin redness, throbbing small
arteries, elevated local temperature,
A. Acute intoxication with narcotic increased tissue turgor. What local
analgesics blood circulation disorder are these
B. Heavy metals intoxication presentations typical of?
C. Intoxication with cardiac glycosides
D. Intoxication with ergot alkaloids A. Arterial hyperemia
E. Atropine sulfate intoxication B. Venous hyperemia
C. Thrombosis
22. Thermodynamic calculations D. Embolism
allow us to determine the possibi- E. Ischemia
lity and direction of spontaneous
processes. In an isolated system 27. Racemose clusters of calci-
the change of the following um carbonate crystals are detected
thermodynamic function is used for among the waste products of a
this purpose: protoplast. These crystals are:
A. Entropy A. Cystoliths
B. Gibbs energy B. Isolated crystals
C. Helmholtz energy C. Raphides
D. Internal energy D. Styloids
E. Enthalpy E. Druses
23. What drug should be administered 28. Purine ring biosynthesis occurs
for individual prevention of malaria? in ribose-5-phosphate by gradual
192
krok123.in.ua
Кrok 1 Pharmacy (англомовнi студенти) 2016 рiк 4
193
krok123.in.ua
Кrok 1 Pharmacy (англомовнi студенти) 2016 рiк 5
C.
197
krok123.in.ua
Кrok 1 Pharmacy (англомовнi студенти) 2016 рiк 9
A. A. N H2 N H2
B. N H2 OH
C. CH3 N H2
D. C6 H5 N HN H2
E. N H3
B.
68. Select the compound with
amphoteric properties (which reacts
both with acids and bases and
produces salts):
C. C6 H5 − N = O
D. C6 H5 − N H − C(O) − CH3 A.
E. (CH3 )2 N − N = O
64. A pharmaceutical enterprise
produces a tetanus-specific preventi-
ve drug. Which drug of those listed B.
below is it?
A. Anatoxin
B. Dead vaccine
C. Live vaccine C.
D. Immunoglobulin
E. Recombinant vaccine
65. Inoculation in a nutrient medi-
um was performed to determine
probable contamination of a drug D.
with fungi. The colonies are large,
resembling sour cream. What nutrient
medium had been used in this case?
A. Sabouraud
B. Loewenstein-Jensen medium E.
C. Roux
D. Loeffler
E. FINN-II
66. Aniline can be converted into the
water-soluble salt, if processed with 69. Select the formula of pentene-2
the solution of: from the list:
67. Specify the reagent necessary for 70. The end product of starch
the following transformation: hydrolysis is:
A. D-glucose
B. D-fructose
C. Saccharose
D. Maltose
E. D-galactose
198
krok123.in.ua
Кrok 1 Pharmacy (англомовнi студенти) 2016 рiк 10
A. B.
B.
C.
D.
E.
E.
72. What reagent can help distinguish
between starch and glucose?
A. I2
B. Br2
C. KMnO4
D. K2 Cr2O7
E. F eCl3 74. What reagent allows to si-
multaneously detect the presence
73. Select the formula, where carbon of both aldehyde group and glycol
atoms numbering complies with fragment in glucose molecule?
IUPAC replacement nomenclature:
A. Cu(OH)2
B. Br2
C. AlCl3
D. F eCl3
E. KMnO4
75. What class of organic compounds
is characterized by the presence of
C ≡ N group?
A. Nitriles
B. Amines
C. Nitro compounds
D. Alcohols
E. Aldehydes
199
krok123.in.ua
Кrok 1 Pharmacy (англомовнi студенти) 2016 рiк 11
203
krok123.in.ua
Кrok 1 Pharmacy (англомовнi студенти) 2016 рiк 15
207
krok123.in.ua
Кrok 1 Pharmacy (англомовнi студенти) 2016 рiк 19
A. Intravenous A. Amlodipine
B. Intramuscular B. Atenolol
C. Subcutaneous C. Guanethidine
D. Internal D. Reserpine
E. Inhalational E. Labetalol
149. What drug should be admini- 154. Colored or white component of
stered in case of acute cardiac insuffi- double perianth, which consists of
ciency? petals, is a:
A. Corglycon A. Corolla
B. Salbutamol B. Flower cup
C. Pilocarpine hydrochloride C. Androecium
D. Naloxone D. Gynoecium
E. Heparin E. Perigonium
150. A patient with signs of mercury 155. Weeds can be harmful for
poisoning has been delivered into populace’s wellbeing. Particularly,
an admission room. What antidote allergic reactions are often caused by
should be prescribed in this case? the following plant in its period of
blossoming:
A. Unithiol
B. Atropine sulfate A. Ambrosia artemisiifolia
C. Proserin B. Equisetum arvense
D. Naloxone C. Stellaria media
E. Calcium chloride D. Erigeron canadensis
E. Taraxacum officinale
151. Diuretic should be prescribed to
treat cerebral edema. What drug is to 156. Aurococcus culture was obtained
be administered? from the nasal cavity of a child sufferi-
ng from chronic tonsillitis. Causative
A. Furosemide agent’s sensitivity towards a number
B. Hydrochlorothiazide of antibiotics was tested to choose the
C. Caffeine and sodium benzoate optimal drug. What drug WAS NOT
D. Diacarb (Acetazolamide) included in antibiotic susceptibility
E. Spironolactone testing?
152. A doctor has prescribed a A. Nystatin
nonsteroidal anti-inflammatory drug B. Ampicillin
to relieve inflammation and pain C. Tetracycline
syndrome. Name this drug: D. Levomycetin (Chloramphenicol)
A. Diclofenac sodium E. Erythromycin
B. Glibenclamide 157. Interferons are natural antiviral
C. Loratadine and antitumor agents. What is their
D. Prednisolone mechanism of action?
E. Calcium chloride
A. Protein synthesis depression
153. The 55-year-old patient has B. Protein synthesis increase
been diagnosed with angina pectoris. C. Replication activation
Calcium channel-blocking agent was D. Transcription activation
prescribed for treatment. Name this E. Repair activation
agent:
158. A patient of a neurology unit
suffers from paralysis of all limbs.
Name this condition:
208
krok123.in.ua
Кrok 1 Pharmacy (англомовнi студенти) 2016 рiк 20
213
krok123.in.ua
Krok 1 Pharmacy (англомовнi студенти) 2017 рiк 1
214
krok123.in.ua
Krok 1 Pharmacy (англомовнi студенти) 2017 рiк 2
216
krok123.in.ua
Krok 1 Pharmacy (англомовнi студенти) 2017 рiк 4
A. 6
B. 4
C. 10
D. 2
E. 8
A.
32. An oncological patient was prescri-
bed fluorouracil that is a competitive
inhibitor of thymidine synthase. It inhi-
bits the process of:
218
krok123.in.ua
Krok 1 Pharmacy (англомовнi студенти) 2017 рiк 6
A.
A. B.
C.
B.
D.
C.
E.
D.
C. C6 H5 − N = O
D. C6 H5 − NH − C(O) − CH3
E. (CH3 )2 N − N = O
49. A patient was delivered into a resusci-
tation unit with signs of alcohol poisoni-
ng. The patient developed hypoxia of the
following pathogenesis:
A. Tissue
B. Hypoxic
C. Hemic
D. Circulatory
E. Mixed
50. Select the compound with
amphoteric properties (which reacts both
with acids and bases and produces salts):
219
krok123.in.ua
Krok 1 Pharmacy (англомовнi студенти) 2017 рiк 7
was administered corglycon. What effect A. The investigated substance does not
of this drug results in improvement of the interact with the titrant
patient’s condition? B. The reaction runs very quickly
C. It is impossible to select the indicator
A. Increased heart force to determine titration end point
B. Decreased heart force D. Side reactions are possible
C. Coronary vessels dilatation E. The reaction runs slowly
D. Increased heart rate
E. Decreased oxygen demand of 108. Colored or white component of
myocardium double perianth, which consists of petals,
is a:
103. Explain to an intern, what is
the mechanism of analgesic action of A. Corolla
morphine hydrochloride: B. Flower cup
C. Androecium
A. Opiate receptors stimulation D. Gynoecium
B. Histamine receptors blockade E. Perigonium
C. Phosphodiesterase blockade
D. Adenylate cyclase stimulation 109. A 54-year-old man requested a
E. Choline esterase blockade pharmacist’s advice on drug prescripti-
on. The patient has 4-year-long history of
104. What pharmacological effect of di- chronic glomerulonephritis and 2-year-
azepam allows its application for termi- long history of persistent hypertension.
nation of convulsions? What substance synthesized in the ki-
dneys has important role in development
A. Anticonvulsant of arterial hypertension?
B. Analgesic
C. Antipyretic A. Renin
D. Anti-inflammatory B. Nitric oxide
E. Hypnotic C. Aldosterone
D. Histamine
105. A patient suffers from hyperchromic E. Catecholamines
B12 -deficiency anemia. What vitamin
preparation should be prescribed in this 110. A patient complains of general
case? weakness, muscle weakness in the
extremities (if the patient is asked to
A. Cyanocobalamin make a fist several times in a row, for
B. Riboflavin example, the patient is capable of doing
C. Vicasol (Menadione) it only once), facial muscles are weak,
D. Thiamine chloride swallowing is disturbed. Administration
E. Retinol acetate of acetylcholinesterase drugs removes
106. A man is diagnosed with Parkinson’s these disturbances to a certain degree.
disease. What drug should be prescribed Determine the pathology:
in this case? A. Myasthenia
A. Levodopa B. Paralysis
B. Nitrazepam C. Paresis
C. Paracetamol D. Hemiplegia
D. Aminazine E. Monoplegia
E. Anaprilin (Propranolol) 111. A drug solution sterilized by
107. Direct titration CANNOT be means of boiling was tested for sterili-
applied for quantitative determinati- ty. Inoculation on Kitt-Tarozzi medium
on of calcium chloride by means of revealed clostridia. Clostridia survived
permanganatometry, because: the boiling because they are:
A. Spore-formers
B. Thermophilic
C. Anaerobic
D. Prototrophic
E. Acid-fast
225
krok123.in.ua
Krok 1 Pharmacy (англомовнi студенти) 2017 рiк 13
229
krok123.in.ua
Krok 1 Pharmacy (англомовнi студенти) 2017 рiк 17
B.
B.
C.
D.
C.
E.
D.
A. A. Hydrogen bonds
B. Ionic bonds
C. Polar covalent bonds
D. Nonpolar covalent bonds
E. Donor-acceptor bonds
B.
192. Specify the functional group of
isoniazide (an isonicotinic acid derivati-
ve) molecule:
C.
D.
A. Hydrazide group
B. Amide group
C. Carboxyl group
E. D. Pyridine heterocyclic group
E. Ester group
193. What compound is synthesized by
189. Essential oils are used both in means of beta-picoline oxidation?
pharmaceutical and cosmetic industry.
To extract essential oils from herbal
raw material the following technology is
used:
A. Steam distillation A. Nicotinic acid
B. Calorimetry B. Benzoic acid
C. Colorimetry C. Uric acid
D. Potentiometry D. Barbituric acid
E. Conductometry E. Ascorbic acid
190. Specify the mechanism of the given 194. Specify quinoline among the
reaction: compounds given below:
A.
+ o
H , t>170 C
C2 H5 OH −−−−−−−−→ H2 C = CH2 +H2 O
B.
A. Е (elimination)
B. SN (nucleophilic substitution) C.
C. N (nucleophilic attachment)
D. SR (radical substitution)
E. S (electrophilic substitution) D.
191. What type of bonds participates
in creation of both linear and cyclic
carboxylic acid associates in the form of
dimers? E.
ng transformation: A.
B.
C.
2+
HOH,Hg [0]
CH ≡ CH −−−−−−−→ A −→ B
D.
E.
B. CH3 − CH − CH2 OH
C. CH3 − CH2 OH A. 6 5 J
D. B. C6 5
C. 6 6
D. 6 5 − Cl
E. C6 5 K
200. What reagent can simultaneously
E. detect aldehyde group and glycol
fragment in a glucose molecule?
A. Cu(OH)2
B. Br2
197. Among the given isomer pairs C. AlCl3
choose the position isomers of the functi- D. F eCl3
onal group: E. KMnO4
235
krok123.in.ua
Krok 1 Pharmacy (англомовний варiант, iноземнi студенти) 2018 рiк 1
A. A.
B. B.
C. C.
D.
D.
E.
E.
240
krok123.in.ua
Krok 1 Pharmacy (англомовний варiант, iноземнi студенти) 2018 рiк 6
241
krok123.in.ua
Krok 1 Pharmacy (англомовний варiант, iноземнi студенти) 2018 рiк 7
242
krok123.in.ua
Krok 1 Pharmacy (англомовний варiант, iноземнi студенти) 2018 рiк 8
70. Choose the reaction of ester production 74. Specify what method of redox titration
among those listed below: requires the use of specific indicator - starch
- to fix the end point:
A.
A. Iodometry
B. Permanganatometry
C. Nitritometry
D. Cerimetry
E. Bromatometry
B.
75. Causative agents of infectious diseases
can be carried both by humans and animals.
Name the group of infections that affect ani-
mals and can be passed onto humans:
C.
A. Zooanthroponoses
B. Sapronoses
C. Anthroponoses
D. Zoonoses
D. E. Mixed
76. During regular check-up a patient
presents with enlarged thyroid gland,
exophthalmos, increased body temperature,
E. heart rate up to 110/min. What hormone
should be measured in the patient’s blood
in this case?
A. Thyroxin
B. Testosterone
C. Glucagon
71. A 25-year-old man has an appointment D. Insulin
with the dentist. Several minutes after his E. Cortisol
oral cavity was lavaged with furacilin (ni-
243
krok123.in.ua
Krok 1 Pharmacy (англомовний варiант, iноземнi студенти) 2018 рiк 9
77. A fruit tree of Rosaceae family has short conclusion additionally, a leaf of this plant
thorny shoots; the fruit is a distinctively- was examined under the microscope and a
shaped pome with stone cells in its pulp. search for the following crystalline inclusi-
Name this plant: ons was conducted:
A. Pyrus communis A. Raphides
B. Malus sylvestris B. Single crystals
C. Cerasus vulgaris C. Druse crystals
D. Prunus armeniaca D. Styloid crystals
E. Prunus spinosa E. Crystal sand
78. Pharmacopoeia reaction to determine 84. What type of conducting bundle
benzoate ions requires interaction with the is characteristic of primary anatomical
solution of: structure of a root?
A. Iron(III) chloride A. Radial
B. Potassium chloride B. Concentric
C. Resorcin C. Closed collateral
D. Acetic anhydride D. Bicollateral
E. Diphenylamine E. Open collateral
79. What solution can be used to detrmine 85. Calendula officinalis as a representative
the presence of chloride ions in the potable of Asteraceae family can be characterized
water? by the following type of inflorescence:
A. Silver nitrate A. Anthodium
B. Iodine B. Umbel
C. Potassium bromate C. Catkin
D. Sodium hydroxide D. Capitulum
E. Ammonia E. Corymb
80. Insulin production in β -cells involves 86. In the age of 5 months the child had
many substances. What substance gives the measles antibodies in the blood. By the age
main signal for insulin synthesis when its of 1 year these antibodies disappeared from
concentration changes? the child’s blood. Why were these antibodi-
es present in the child’s blood?
A. Glucose
B. Carbon dioxide A. Acquired natural passive immunity
C. Heparin B. Non-specific resistance
D. Hemoglobin C. Acquired natural active immunity
E. Urea D. Innate immunity
E. Artificial immunity
81. Specify the substance that results from
the following reaction: 87. A Gram stained smear shows large
HOH, Hg 2+ oval violet cells that form pseudomycelium.
CH ≡ CH −−−−−−−→? Name these microorganisms:
A. Ethanal A. Candida fungi
B. Ethanol B. Mucor fungi
C. Propanal C. Plasmodium vivax
D. Propanone D. Actinomycetales
E. Acetic acid E. Penicillium fungi
82. An ophthalmologist has detected 88. Herbarium specimens of medicinal
increased time of dark adaptation in a pati- plants are being studied. Which one of them
ent. What vitamin deficiency can result in belongs to Rosaceae family?
such symptom?
A. Crataegus sanguinea
A. A B. Melilotus officinalis
B. C C. Conium maculatum
C. K D. Capsella bursa-pastoris
D. B1 E. Polygonum persicaria
E. B6
89. When working in the garden, a man
83. Morphologically the herbaceous accidentally cut his hand. The wound
plant being studied can be identified remained untreated. Shortly after that the
as Convallaria majalis. To confirm this
244
krok123.in.ua
Krok 1 Pharmacy (англомовний варiант, iноземнi студенти) 2018 рiк 10
250
krok123.in.ua
Krok 1 Pharmacy (англомовний варiант, iноземнi студенти) 2018 рiк 16
C.
D.
E.
251
krok123.in.ua
Krok 1 Pharmacy (англомовний варiант, iноземнi студенти) 2018 рiк 17
A. A. Rimantadine
B. Methisazone
C. Levamisole
D. Idoxuridine
E. Doxycycline
252
krok123.in.ua
Krok 1 Pharmacy (англомовний варiант, iноземнi студенти) 2018 рiк 18
A. Interferon
B. Benzoteph A.
C. Carvedilol
D. Tetracycline
E. Doxorubicin
175. A patient came to the pharmacy to
obtain an antidiarrheal agent. What drug B.
would be recommended by the dispensing
chemist?
A. Loperamide
B. Dicaine (Tetracaine) C.
C. Ranitidine
D. Picolax (Sodium picosulfate)
E. Anesthesin (Benzocaine)
176. A patient came to the pharmacy to
obtain a drug that contains pancreatic D.
enzymes and can be taked for chronic
pancreatitis. What drug would be
recommended by the dispensing chemist?
A. Pancreatine
B. Triamcinolone
C. Gordox (Aprotinin) E.
D. Pirenzepine
E. Omeprazole
177. Increased concentration of acti-
ve oxygen forms is a mechanism of
pathogenesis in a number of diseases.
To prevent this process, antioxidants are 181. When an isolated system spontaneously
prescribed. Select an antioxidant from the approaches its equilibrium, its entropy:
list below:
A. Approaches zero
A. α-tocopherol B. Approaches infinity
B. Glucose C. Reaches maximum
C. Calciferol D. Reaches minimum
D. Cobalamine E. Demonstrates linear magnification
E. Glicerol
182. A pharmacy produces a batch of vials
178. A 28-year-old man with peptic ulcer of with physiological saline for injections. How
the stomach was prescribed a drug that inhi- should they be sterilized?
bits gastric juice secretion. Specify this drug:
A. Under pressure in an autoclave
A. Omeprazole B. In a steam-jacketed autoclave chamber
B. Ethacrynic acid C. In a dry heat sterilizer
C. Duphalac (Lactulose) D. X-ray irradiation
D. Lidocaine E. Ultraviolet irradiation
E. Fenofibrate
183. A certain dioecious plant commonly
179. An engine driver complains of his grows at the forest edge. It is a shrub with
seasonal allergy symptoms. What non- thorned sprouts. Its fruit is a round black
sedating drug should be prescribed in this coenocarpous drupe (pyrenarium) with 3-4
case? seeds. Name this plant:
A. Loratadine A. Rhamnus cathartica
B. Novocaine B. Hippophae rhamnoides
C. Fenofibrate C. Crataegus sanguinea
D. Analgine (Metamizole) D. Rosa canina
E. Atenolol E. Sambucus nigra
180. Salicylic acid derivatives are widely 184. Choose the weakest carboxylic acid
used in medicine. Specify the formula of basing on its pKa value:
salicylic acid:
253
krok123.in.ua
Krok 1 Pharmacy (англомовний варiант, iноземнi студенти) 2018 рiк 19
185. Which of the amines listed below is a 191. Specify the analgesic that affects opi-
primary amine? ate receptors and can cause development of
tolerance and dependence:
A. C6 H5 CH2 NH2
B. C6 H5 CH2 NHCH3 A. Morphine
C. C6 H5 NHCH3 B. Phenobarbital
D. C6 H5 N(CH3 )2 C. Medazepam
E. C6 H5 CH2 N(CH3 )2 D. Voltaren (Diclofenac sodium)
E. Haloperidol
186. It can be safely assumed that the infants
born from the mothers with the history of 192. During furosemide therapy of a pati-
measles will not be affected by the measles ent with chronic edematous syndrome, his
outbreak during their stay in the materni- plasma-cation concentration was disturbed.
ty ward. What classes of antibodies provide What drug should be used in this case?
the infants with the resistance to this di- A. Potassium chloride
sease? B. Thiamine bromide
A. IgG C. Ascorutin (Ascorbic acid + Rutoside)
B. IgA D. Magne B6
C. IgD E. Aspirin
D. IgM 193. Upon examination the ophthalmologi-
E. IgE st diagnosed a 21-year-old woman with vi-
187. The children attending a kindergarten sual impairment - hemeralopia (”night bli-
were hospitalized with diagnosis of poli- ndness”). What drug should this patient
omyelitis. What was the route of infection take to restore her vision?
transmission in this case?
A. Retinol acetate
A. Fecal-oral transmission B. Ergocalciferol
B. Alimentary transmission C. Suprastin (Chloropyramine)
C. Direct contact transmission D. Cholecalciferol
D. Transmission via airborne dust particles E. Sustac forte (Nitroglycerin)
E. Vector-borne transmission
194. What bonds participate in creation of
188. What ion increases osmotic pressure in both linear and cyclic dimeric carboxylic
the focus of inflammation? acid associates?
A. Potassium
B. Calcium
C. Fluorine
D. Magnesium
E. Chlorine A. Hydrogen bonds
B. Ionic bonds
189. A 55-year-old man came to a doctor wi- C. Polar covalent bonds
th complaints of acute pain in his big toes. D. Nonpolar covalent bonds
Meat and wine are a permanent fixture in E. Donor-acceptor bonds
his diet. The doctor suspects gout. What
substance must be measured in the patient’s 195. Heating of sodium phenolate in
blood to confirm this diagnosis? CO2 stream results in production of a
certain carboxylic acid. Name the resulti-
A. Uric acid ng compound:
B. Urea
C. Lactate A. Salicylic acid
D. Bilirubin B. Ethyl salicylate
E. Ketone bodies C. Phenyl salicylate
D. Benzoic acid
190. Name the process of cell membrane E. Aminophenol
saturation with a fat-like substance -
suberin: 196. Which alkadiene of those listed below
254
krok123.in.ua
Krok 1 Pharmacy (англомовний варiант, iноземнi студенти) 2018 рiк 20
A. Nicotinic acid
B. B. Benzoic acid
C. Uric acid
D. Barbituric acid
E. Ascorbic acid
200. Specify quinoline among the
compounds given below:
C.
A.
B.
D.
C.
E. D.
E.
255
krok123.in.ua